Решение уравнений по модулю: Решение уравнений с модулями

Содержание

Решение уравнения с модулем

Решение уравнений с модулем. В этой статье я покажу алгоритм решения уравнений, которые содержат  несколько выражений под знаком модуля, на примере решения уравнения уровня С1, а затем вы посмотрите ВИДЕОУРОК с подробным разбором тригонометрического уравнения с модулем.

Давайте решим уравнение:


 

Вспомним, что модуль раскрывается по  такому правилу:

Говоря человеческим языком, модуль выражения равен самому выражению, если оно неотрицательно, и выражению с противоположным знаком, если оно меньше нуля.

Таким образом, перед нами стоит задача раскрыть все модули в соответствии со знаками подмодульных выражений.

Будем следовать такому алгоритму:

1. Определим, в каких точках каждое подмодульное выражение меняет знак. Для этого приравняем каждое подмодульное выражение к нулю:

Мы получили три точки.

2. Нанесем их на числовую ось:

Эти три числа разбили числовую ось на четыре промежутка:

,   ,   ,  

Обратите внимание, что мы включили крайние точки промежутков в оба промежутка. Ничего страшного не случится, если мы эти точки учтем два раза, главное, о них не забыть.

3. Теперь рассмотрим знаки подмодульных выражений на каждом промежутке:

Выражение меняет знак в точке . Слева от этой точки оно отрицательно, а справа положительно. Отметим это в таблице:

Выражение меняет знак в точке . Слева от этой точки оно отрицательно, а справа положительно. Отметим это в таблице:

Выражение меняет знак в точке . Слева от этой точки оно отрицательно, а справа положительно. Отметим это в таблице:

Мы получили знаки всех подмодульных выражений на каждом промежутке. Теперь раскроем модули на каждом промежутке с учетом этих знаков.

Наше уравнение «распадается» на четыре уравнения по количеству числовых промежутков.

4. Решим уравнение на каждом промежутке:

1. 

Решение уравнения на первом промежутке 

2.Раскроем модули на втором промежутке:

Мы получили, что второе уравнение системы является тождеством, то есть   второе равенство верно при любом действительном значении . Следовательно, решением системы будут те значения неизвестного, которые удовлетворяют первому неравенству:

.

3. Раскроем модули на третьем промежутке:

Решение уравнения на третьем промежутке: 

4. Раскроем модули на четвертом промежутке:

Решение уравнения на четвертом промежутке: 

Заметим, что решения нашего уравнения на каждом промежутке принадлежали этому промежутку, то есть удовлетворяли неравенству каждой системы. Однако, так бывает не всегда, и если корень уравнения не удовлетворяет неравенству, значит, соответствующая система не имеет решений.

5. Теперь объединим полученные решения, и запишем ответ:

Ответ: -6≤х≤0, х=12

А сейчас я предлагаю вам посмотреть ВИДЕУРОК с подробным решением уравнения уровня С3:

 

И.В. Фельдман, репетитор по математике.

Решение уравнений с модулем (часть 2)

Уравнения с модулем с решениями (часть 2)

перейти к содержанию

Свойства модуля (справочник)

11.  Найдите среднее арифметическое корней уравнения

Решение

Уравнение равносильно совокупности при условии . Уравнения можно упростить к виду  , откуда , или . Неравенству удовлетворяют только и . Среднее арифметическое корней равно .

Ответ:  

12. Решите уравнение

Решение

Так как подкоренное выражение должно быть неотрицательным, а знаменатель дроби не должен быть равен нулю, то . Рассмотрим два случая: и .

Первом случае числитель равен , то есть все числа из промежутка являются решением исходного уравнения.

Во втором случае . Но . Поэтому корней нет.

Ответ:

13. Решите уравнение

Решение

Так как подкоренное выражение должно быть неотрицательным, а знаменатель дроби не должен быть равен нулю, то . Рассмотрим два случая: и .

Первом случае числитель равен , то есть корней нет.

Во втором случае , то есть все числа из промежутка являются решением исходного уравнения.

Ответ:

14. Найдите сумму корней уравнения

Решение

Уравнение равносильно совокупности . Первое уравнение равносильно , откуда или . Решением второго уравнения является . Сумма корней равна .

Ответ: 

15. Найдите количество целых корней уравнения на отрезке

Решение

. Применим следующее утверждение: . Тогда . Целые числа: .

Ответ: 

16. Найдите количество натуральных корней уравнения

Решение

. Так как , то уравнение равносильно системе неравенств , откуда . Натуральные корни исходного уравнения: .

Ответ: 

17. Определите, при каких значениях параметра уравнение имеет ровно два корня.

Решение

Ответ: 

18. При каких значениях параметра уравнение имеет бесконечно много корней?

Решение

Ответ: 

19. При каких значениях параметра уравнение имеет ровно четыре корня?

Решение

Ответ: 

20. При каких значениях параметра система  имеет более одного решения?

Решение

Ответ: 

смотрите раздел «Математика»

 

Урок 4: Уравнения с модулем

План урока:

Модуль числа

Решение уравнений с модулем

Уравнения с параметрами

 

Модуль числа

Напомним, что такое модуль числа. Так называют значение числа, взятое без учета его знака. То есть модуль чисел 9 и (– 9) одинаков и равен 9. Для обозначения модуля применяют специальные прямоугольные скобки:

|9| = |– 9| = 9

|674| = |– 674| = 674

|2,536| = |– 2,536| = 2,536

Грубо говоря, операция нахождения модуля сводится к отбрасыванию у числа знака «минус», если он у него есть. Вообще, если число х неотрицательно, то его модуль |х| = х. Если же число отрицательно, то его модуль имеет противоположное значение: |х| = х. Математически это можно записать так:

Именно такое определение обычно и применяется в математике.

Модуль играет важную роль в математике. Дело в том, с его помощью удобно записывать расстояние между двумя точками на координатной прямой. Пусть на ней отмечены точки a и b. Расстояние между ними равно |a – b|, причем неважно, какое из этих чисел больше, а какое меньше:

Также модуль возникает при извлечении квадратного корня из четной степени числа:

В частности, если n = 1, получим формулу:

Для того чтобы получить график функции у = |x|, сначала надо построить график функции без учета знака модуля:

Далее следует выполнить преобразование. Те точки графика, которые располагаются выше оси Ох, остаются на своем месте. В данном случае это та часть графика, которая находится в I четверти. Те же точки, которые располагаются ниже оси Ох, должны быть симметрично (относительно этой самой оси Ох) отображены. В результате они окажутся выше оси Ох:

В результате получилась «галочка».

Пример. Постройте график ф-ции у = |х2 – 4х + 3|

Решение. Для построения графика функции, содержащей модуль, сначала надо построить график для «подмодульного» выражения. Поэтому построим график у = х2 – 4х + 3. Это квадратичная ф-ция, ее график – это парабола:

Часть графика, в промежутке от 1 до 3, находится ниже оси Ох. Чтобы построить ф-цию у = |х2 – 4х + 3|, надо перевернуть эту часть графика:

Решение уравнений с модулем

Изучим простейший случай уравнения, содержащего модуль, когда вся его слева записано выр-ние в модульных скобках, а справа находится число. То есть уравнение имеет вид

|у(х)| = b

где b – какое-то число, а у(х) – произвольная ф-ция.

Если b< 0, то ур-ние корней не имеет, ведь модуль не может быть отрицательным.

 

Пример. Найдите корни ур-ния

|125x10 + 97x4– 12,56х3 + 52х2 + 1001х – 1234| = – 15

Решение: Справа стоит отрицательное число. Однако модуль не может быть меньше нуля. Это значит, что у ур-ния отсутствуют корни.

Ответ: корни отсутствуют.

Если b = 0, то мы получим какое-то произвольное ур-ние у(х) = 0, у которого могут быть корни. Проще говоря, модульные скобки в таком случае можно просто убрать.

 

Пример. Решите ур-ние

|13х – 52| = 0

Решение.

Ясно, что подмодульное выр-ние равно нулю:

13х – 52 = 0

13х = 52

х = 4

Ответ: 4.

Наиболее интересен случай, когда b> 0, то есть в правой части стоит положительное число. Ясно, что тогда под модулем находится либо само это число b, либо противоположное ему число – b:

|b| = b

|– b| = b

То есть мы получаем два различных ур-ния: у(х) = bи у(х) = – b.

 

Пример. Решите ур-ние

|х| = 10

Решение. В правой части – положительное число, поэтому либо х = – 10, либо х = 10.

Ответ: 10; (– 10).

 

Пример. Решите ур-ние

|10х + 5| = 7

Решение. Исходное ур-ние разбивается на два других ур-ния:

10х + 5 = 7 или 10х + 5 = – 7

10х = 2 или 10х = – 12

х = 0,2 или х = – 1,2

Ответ: 0,2; (– 1,2).

 

Пример. Найдите корни ур-ния

|x2– 2х – 4| = 4

Решение. Снова заменим исходное равенство на два других:

x2– 2х – 4 = 4 или x2– 2х – 4 = – 4

Имеем два квадратных ур-ния. Решим каждое из них:

x2– 2х – 4 = 4

x2– 2х – 8 = 0

D = b2– 4ас = (– 2)2 – 4•1•(– 8) = 4 + 32 = 36

х1 = (2 – 6)/2 = – 2

х2 = (2 + 6)/2 = 4

Нашли корни (– 2) и 4. Решаем второе ур-ние:

x2– 2х – 4 = – 4

x2– 2х = 0

х(х – 2) = 0

х = 0 или х – 2 = 0

х = 0 или х = 2

Получили ещё два корня: 0 и 2.

Ответ: – 2, 4, 0, 2

Встречаются случаи, когда в уравнении, содержащем знак модуля, под ним находятся обе части равенства:

|у(х)| = |g(x)|

Здесь возможны два варианта. Либо подмодульные выр-ния равны друг другу (у(х) = g(x)), либо у них противоположные значения (у(х) = – g(x)). То есть снова надо решить два ур-ния.

 

Пример. Решите ур-ние

|x

2 + 2x– 1| = |х + 1|

Решение. Выр-ния справа и слева (без знака модуля) либо равны, либо противоположны. Можно составить два ур-ния:

x2 + 2x– 1 = х + 1 или x2 + 2x– 1 = – (х + 1)

х2 + х – 2 = 0 или х2 + 3х = 0

Решим 1-ое ур-ние:

х2 + х – 2 = 0

D = b2– 4ас = 12 – 4•1•(– 2) = 1 + 8 = 9

х1 = (1 – 3)/2 = – 1

х2 = (1 + 3)/2 = 2

Теперь переходим ко 2-омуур-нию:

х2 + 3х = 0

х(х + 3) = 0

х = 0 или х + 3 = 0

х = 0 или х = – 3

Всего удалось найти 4 корня: (– 1), (– 2), 2 и 0.

Ответ:(– 1), (– 2), 2, 0.

Возможен случай, когда в левой части равенства находится модуль выр-ния, а в правой – обычное выражение, без модуля. Такое ур-ние имеет вид |у(х)| = g(x). Здесь также возможны два варианта: у(х) = g(x) или у(х) = – g(x). Однако следует учитывать ещё один факт. Модуль не может быть отрицательным, а потому должно выполняться нер-во g(x)⩾ 0. Но это неравенство не надо решать. Достаточно просто подставить в него все полученные корни и проверить, справедливо ли нер-во.

Пример. Найдите решение уравнения, содержащего модуль:

2 + 3,5х – 20| = 4,5х

Решение. Рассмотрим два отдельных равенства:

х2 + 3,5х – 20 = 4,5х илих2 + 3,5х – 20 = – 4,5х

х2 – х – 20 = 0 или х2 + 8х – 20 = 0

Решим каждое из полученных квадратных ур-ний.

х2 – х – 20 = 0

D = b2– 4ас = 12 – 4•1•(– 20) = 1 + 80 = 81

х1 = (1 – 9)/2 = – 4

х2 = (1 + 9)/2 = 5

х2 + 8х – 20 = 0

D = b2– 4ас = 82 – 4•1•(– 20) = 64 + 80 = 144

х3 = (– 8 – 12)/2 = – 10

х4 = (– 8 + 12)/2 = 2

Итак, получили 4 корня: (– 4), 5, (– 10) и 2. Однако правая часть исходного ур-ния, 4,5x, не может быть отрицательной, ведь модуль числа – это всегда неотрицательная величина:

4,5х ≥ 0

Для х = – 4 и х = – 10 это условие не выполняется, поэтому эти корни должны быть исключены.

Ответ: 2 и 5

Мы рассмотрели три случая, когда ур-ние имеет вид:

  1. у(х) = b (b– это некоторая константа)
  2. |у(х)| = |g(x)|
  3. |у(х)| = g(x)

Однако порою ур-ние не удается свести ни к одному из этих видов. Тогда для решения уравнений и неравенств, содержащих модуль, следует рассматривать их на отдельных интервалах, где подмодульные выр-ния не изменяют свой знак.

 

Пример. Найдите корни ур-ния

|x + 1| + |x– 4| = 6

Решение. Выр-ния х + 1 и х – 4 меняют знак при переходе через точки (– 1) и 4:

Если отметить обе точки на прямой, то они образуют на ней 3 интервала:

Исследуем ур-ние на каждом из полученных промежутков.

Так как при х <– 1 оба подмодульные выр-ния отрицательны, то можно записать, что

|x + 1| = – (х + 1) = – х – 1

|x– 4| = – (х – 4) = – х + 4

Тогда ур-ние примет вид

|x + 1| + |x– 4| = 6

– х – 1 – х + 4 = 6

–2х = 3

х = – 1,5

Это значение удовлетворяет условию х <– 1, поэтому корень верный.

Далее изучим случай, когда х∊[– 1; 4). Здесь отрицательно только выражение x– 4, поэтому модули заменяются так:

|x + 1| = х + 1

|x– 4| = – (х – 4) = – х + 4

Ур-ние примет вид:

|x + 1| + |x– 4| = 6

x + 1 – x+ 4 = 6

5 = 6

Получили неверное тождество. Получается, что на промежутке [– 1; 4) корней нет.

При х ≥4 выр-ния х – 4 и х + 1 положительны, поэтому

|x + 1| = х + 1

|x– 4| = х – 4

Исходное ур-ние будет выглядеть так

|x + 1| + |x– 4| = 6

х + 1 + х – 4 = 6

2х = 9

х = 4,5

Найденный корень удовлетворяет условию х ≥4, поэтому он также должен быть включен в ответ.

 

Уравнения с параметрами

Изучим ур-ния:

5х = 10

5х = 15

5х = 20

Для решения каждого из них надо число справа поделить на 5 (множитель перед х). В итоге получаем значения х, равные 2, 3 и 4.

Теперь обозначим число в правой части буквой, например, как v. Тогда все эти ур-ния будут выглядеть одинаково:

5х = v

Решением таких ур-ний будет дробь v/5.

Надо понимать разный смысл, который мы вкладываем при этом в буквы х и v. Через х мы обозначили переменную, то есть ту величину, значение которой необходимо найти. Под буквой подразумевалась заранее известная величина, то есть константа, которая известна заранее в каждом конкретном ур-нии. Такую величину называют параметром, а ур-ние 5х = v называют уравнением с параметром.

Изучая уравнение с параметром, мы рассматриваем не одно конкретное ур-ние, а сразу целую группу, или семейство ур-ний. Например, все ур-ния первой степени можно описать в виде

ах + b= 0

где х – это переменная величина, а числа а, b– это параметры. Для описания квадратного ур-ния в общем виде необходимы уже три параметра (а, b и с):

ах2 + bx + c = 0

Параметры встречаются не только при описании ур-ний, но и, например, при рассмотрении функций. Так, линейная функция задается формулой у = kx + b. Здесь числа k и b являются параметрами. Так как ур-ние у = kx + b задает на плоскости прямую линию, то величины k и b порою называют параметрами уравнения прямой.

Если при решении обычного ур-ния мы определяем значение его корней в виде конкретных чисел, то при решении ур-ний с параметром находят формулу, позволяющую при заданном значении параметра вычислить значение корня.

 

Пример. Решите ур-ние

х2 – 2ах = 0

и найдите его корни при значении параметра а, равном 3.

Решение. Вынесем множитель х за скобки:

х2 – 2ах = 0

х(х – 2а) = 0

х = 0 или х – 2а = 0

х = 0 или х = 2а

Получили, что при любом значении параметра а ур-ние имеет два корня. Один из них равен нулю при любом значении а, а второй вычисляется по формуле х = 2а:

при а = 3х = 2•3 = 6

Ответ: есть два корня – 0 и 2а. При а = 2 корни равны 0 и 6.

 

Пример. Решите ур-ние

р2х – 3рх = р2 – 9

Решение. Слева вынесем за скобки множитель рх, а выр-ние справа преобразуем, используя формулу разности квадратов:

рх(р – 3) = (р – 3)(р + 3)

Возникает желание поделить обе части рав-ва на р(р – 3), чтобы выразить х. Однако сразу так делать нельзя, ведь если величина р(р – 3) равна нулю, то получится деление на ноль.

Поэтому сначала изучим случаи, когда один из множителей слева равен нулю. Если р = 0, то мы получим рав-во

0•х•(0 – 3) = (0 – 3) (3 – 0)

0 = – 9

Это неверное тождество, а потому при р = 0 ур-ние корней не имеет.

Если р – 3 = 0, то есть р = 3, получится следующее

3•х•0 = 0•(3 + 3)

0 = 0

Это равенство верно при любом х. Значит, при р = 3 корнем ур-ния является любое число.

Если же р≠ 0 и р ≠ 3, то произведение р(р – 3) также не равно нулю, а потому обе части равенства можно поделить на р(р – 3). Тогда получим

В этом случае ур-ние имеет единственный корень.

Ответ: при р = 0 корней нет; при р = 3 корнем является любое число; при других рх = (р + 3)/р.

Часто в задаче требуется не выразить корень ур-ния через параметр, а лишь оценить количество корней ур-ния или диапазон их значений.

 

Пример. Сколько корней имеет ур-ние

2 – 6х + 5| = b

при различных значениях параметра b.

Решение. Будем решать ур-ние графическим методом. Для этого сначала построим график у = |х2 – 6х + 5|. В модульных скобках находится обычная квадратичная функция, чьи ветви смотрят вверх. Найдем нули функции:

х2 – 6х + 5 = 0

D = b2– 4ас = (– 6)2 – 4•1•5 = 36 + 20 = 16

х1 = (6 – 4)/2 = 1

х2 = (6 + 4)/2 = 5

Итак, нули ф-ции – это точки 1 и 5. Найдем координату х0 вершины параболы по формуле:

х0 = –b/2a = 6/2 = 3

Подставив х0 в квадратичную ф-цию найдем координату у0 вершины параболы:

32 – 6•3 + 5 = 9 – 18 + 5 = – 4

Теперь построим квадратичную ф-цию:

Для построения графика, содержащего модуль функции, надо отобразить точки с отрицательными ординатами (они находятся ниже оси Ох) симметрично относительно оси Ох:

Мы построили график левой части ур-ния. График правой части представляет собой горизонтальную прямую у = b. Можно выделить 5 различных случаев взаимного расположения этих графиков:

При b< 0 прямая пролегает ниже графика. Общих точек у графиков нет, а потому ур-ние корней не имеет.

При b = 0 прямая у = 0 касается графика в 2 точках: (1; 0) и (5; 0). Получаем 2 корня.

Если 0 <b< 4, то прямая пересекает график в 4 точках.

При b = 4 прямая у = 4 касается перевернутой вершины параболы, а также пересекает ветви ещё в 2 точках. Итого 3 корня.

Наконец, при b>4 есть горизонтальная прямая пересекает график лишь в 2 точках, то есть получаем 2 корня.

Ответ: нет корней при b< 0; 2 корня при b = 0 и b> 4; 3 корня при b = 4; 4 корня при 0 <b< 4.

Пример. При каком а ур-ние

х4 – (а + 2)х2 + 3а – 3 = 0

имеет ровно 4 корня?

Решение. Это ур-ние является биквадратным, то есть для его решения нужно произвести замену у = х2:

у2 – (а + 2)у + 3а – 3 = 0 (1)

Для того, чтобы исходное ур-ние имело 4 корня, необходимо, чтобы у квадратного уравнения с параметром(1) было два положительных корня: у1 и у2. Тогда, проводя обратную замену х2 = у1 и х2 = у2, мы получим два разных квадратных ур-ния, корни которых будут равны

Если же хоть один из двух корней, например, у1, окажется равным нулю, то величины

Совпадут (они обе будут равны нулю), и останется лишь 3 корня. Если же у1 будет отрицательным числом, то ур-ние

х2 = у1

вовсе не будет иметь решений, и тогда останется не более 2 корней.

Итак, решим ур-ние (1):

у2 – (а + 2)у + 3а – 3 = 0

D = b2– 4ас = (– (а + 2))2 – 4•1•(3а – 3) = (а + 2)2 – 12 а + 12 =

= а2 + 4а + 4 – 12а + 12 = а2 – 8а + 16 = а2 – 2•4•а + 42 = (а – 4)2

Чтобы у ур-ния (1) было два различных корня, дискриминант должен быть положительным. Величина (а – 4)2 положительна при всех значениях а, кроме а = 4, которое обращает дискриминант в ноль. Значит, а ≠ 4.

Извлечем корень из дискриминанта:

Корни ур-ния (1) можно вычислить по формулам:

И у1, и у2 должны быть положительными величинами, однако у1 меньше, чем у2 (ведь для его вычисления дискриминант брали со знаком «минус», а не «плюс»). Поэтому достаточно записать нер-во:

Получили неравенство, содержащее модуль. Для избавления от модульных скобок в нер-ве рассмотрим 2 случая. Если а – 4>0, то есть а > 4, выполняется равенство

|а – 4| = а – 4

Тогда имеем

а + 2 – (а – 4) > 0

6> 0

Это нер-во выполняется при любом допустимом значении а, поэтому при а >4 исходное ур-ние имеет 4 корня.

Если а < 4, то справедливо соотношение

|а – 4| = – (а – 4)

Тогда получится следующее:

а + 2 – |а – 4|> 0

а + 2 – (– (а – 4)) > 0

а + 2 + а – 4 > 0

2а > 2

а > 1

Итак, при условии, что а< 4, должно выполняться нер-во а > 1. Это значит, что а∊(1; 4). С учетом первого случая, при котором было получено решение

а > 4

можно записать окончательный ответ: а∊(1; 4)∪(4; + ∞).

Ответ: а∊(1; 4)∪(4; + ∞).

 

Пример. При каких параметрах а у ур-ния

х2 – 2(а + 1)х + а2 + 2а – 3 = 0

существует два корня, которые принадлежат интервалу (– 5; 5)?

Решение. Данное ур-ние является квадратным. Найдем его дискриминант:

D = b2– 4ас = (– 2(а + 1))2 – 4•1•( а2 + 2а – 3) = 4(а2 + 2а + 1) – 4(а2 + 2а – 3) =

= 4(а2 + 2а + 1 – а2– 2а + 3) = 4•4 = 16

Получаем, что при любом а дискриминант положителен, а потому уур-ния 2 корня. Вычислить их можно по формулам

Для того, чтобы оба решения уравнения с параметром принадлежали интервалу (– 5; 5), нужно, чтобы меньший из них (это х1) был больше – 5, больший (это х2) – меньше – 5:

Значит, должны выполняться два нер-ва

х1>– 5и х2<5

а – 1 >– 5 и а + 3 < 5

а >– 4 и а < 2

Эти два нер-ва выполняются, если а∊(– 4; 2)

Ответ: (– 4; 2)

 

«Практикум решения уравнений с модулем»

Аттестационная работа слушателя курсов повышения квалификации по программе: «Проектная и исследовательская деятельность как способ формирования метапредметных результатов обучения в условиях реализации ФГОС»

Кугушева Наталья Ивановна

КГБ ПОУ «Минусинский сельскохозяйственный колледж» г. Минусинск, Красноярский край.

На тему:

Методическая разработка

«Практикум решения уравнений с модулем»

Краткая характеристика методической разработки

  • Данная разработка может быть предложена обучающимся 9-11 классов, студентам 1 курса средне-специальных учреждений в рамках урочной деятельности и элективного курса.
  • Изучение материала построено по принципу «от простого к сложному». В начале рассматриваются задания на преобразование выражений, содержащих модуль, затем простейшие уравнения с модулем.
  • В рамках изучения темы, рассматриваются следующие вопросы: определение и свойства модуля; преобразования выражений, содержащих модуль; решение простейших уравнений с модулем; общие методы решения уравнений с модулем; метод интервалов.

Краткая характеристика КГБ ПОУ «Минусинского сельскохозяйственного колледжа»

  • Колледж готовит студентов по специальностям: Механизация сельского хозяйства; Электрификация и автоматизация сельского хозяйства; Теплоснабжение и теплотехническое оборудование; Экономика и бухгалтерский учет; Технология хлебопечения, кондитерских и макаронных изделий.
  • Отделение, на котором я работаю, образовалось при объединении колледжа с сельским профессиональным училищем, и готовит для сельского хозяйства рабочих по профессиям: тракторист-машинист сельскохозяйственного производства; повар-кондитер;

портной. Срок обучения на данном отделении 2года 10 месяцев.

Цель и задачи методической разработки

Цель: обобщение и систематизация знаний, связанных с определением и свойствами модуля.

Задачи:

  • Повысить интерес к математике за счет дифференцированного подхода к решению математических заданий с модулем.
  • Способствовать развитию практического опыта решения уравнений, неравенств и систем уравнений и неравенств, содержащих модули.
  • Помочь овладеть рядом технических и интеллектуальных умений на уровне свободного их применения.
  • Формировать умение работать со справочной литературой, находить и использовать информацию в рекомендованных изданиях.

Формы работы

В процессе изучения темы «Решение уравнений с модулем» учащиеся могут включиться в такие виды деятельности, как:

  • поиск и анализ необходимой информации, в том числе с помощью Интернета;
  • работа в группах при составлении и решении заданий;
  • устные выступления по способам решения с последующей дискуссией;
  • оформление результатов деятельности в форме набора уравнений и неравенств, а также их систем или компьютерной презентации.

Основное содержание работы

Задание 1. Вспомнить или найти в справочной литературе определение модуля.

Модулем (абсолютной величиной) действительного числа называется само это число, если a≥ 0, и противоположное число – a , если a

Основное содержание работы

Задание 2. Работа в группе. Найдите и запишите свойства модуля.

  • |a|≥0
  • |a|=|-a|
  • |a·b|=|a|·|b|
  • |a:b|=|a|:|b|
  • |a|²=a²
  • |a-b| есть расстояние между точками a и b числовой оси.

Основное содержание работы

Презентация основных способов решения уравнений с модулем.

  • По определению.
  • Возведение обеих частей уравнения в квадрат.
  • Замена переменной.
  • «Раскрытие» модуля на промежутке знакопостоянства.
  • Использования геометрического смысла модуля.

Основное содержание работы

Задание 3. Выбрать, к какому способу решения уравнений с модулем относится каждый из предложенных алгоритмов.

Алгоритм 1.

1. Решить уравнение | а | = а.

2. Решить уравнение | а | = -а.

3. Сделать проверку найденных корней.

4. Записать ответ.

Алгоритм 2.

1. Обозначить | х | = t .

2. Решить полученное уравнение относительно t .

3. Сделать замену на х.

4. Сделать проверку найденных значений х.

5. Записать ответ.

Основное содержание работы

Алгоритм 3.

1. Возвести левую и правую части уравнения в квадрат.

2. В полученном равносильном уравнении найти корни.

3. Сделать проверку. Записать ответ.

Алгоритм 4.

1. Найти нули всех подмодульных выражений,

расположить их по мере возрастания на числовой оси.

2. На полученных интервалах определить знак всех

подмодульных выражений и раскрыть модули по

определению.

3. Найти решение уравнения на каждом интервале.

4. Объединить эти решения. Записать ответ.

Основное содержание работы

Задание 4. Выясните, к какому способу

решения уравнений с модулем алгоритм не

предложен. Найдите к этому способу

соответствующие примеры. Составьте

алгоритм решения.

Задание 5. Подберите по 3 примера к каждому

из рассмотренных способов. Решите их,

объясняя каждый этап решения.

  • Гайдуков И.И. Абсолютная величина. Пособие для учителей.Изд.2-е.М. «Просвещение»,1995.
  • Семенко Е.А. Готовимся к ЕГЭ. Обобщающее повторение курса алгебры и начала анализа. Краснодар: «Просвещение — Юг»,2005, 1 часть.
  • Сканави М.И. Сборник задач по математике для поступающих в вузы. М.: ООО «Издательский дом» ОНИКС 21 век.2003.
  • Никольский С.М. Алгебра и начала анализа. Изд. «Просвещение», 2009.
  • Под ред.Фальке Л.Я.Изучение сложных тем курса алгебры в средней школе. Пособие по математике. Изд.2-е.- М.: Народное образование; Илекса; Ставрополь: Сервисшкола, 2004.

Методы диагностики образовательных результатов

Если учащийся:

  • принимал активное участие в практикумах,
  • успешно выполнил индивидуальные домашние

задания,

  • продемонстрировал умение использовать

справочную литературу,

  • научился работать в группах,
  • находить и использовать информацию в

рекомендованных изданиях.

То он набирает от 30 до 50 баллов (максимальное

количество 50 баллов) и получает зачет. Каждое

задание 10 баллов.

Перспективы развития исследовательской деятельности в профессиональной работе

1. Систематизация имеющегося материала по темам.

Корректировка групповых и домашних заданий,

используя приемы исследовательской деятельности.

2. Презентация образовательных результатов учащихся

на дистанционных и очных олимпиадах по математике

разного уровня (очная межрегиональная олимпиада

«Ищем Ломоносовых», международная дистанционная

олимпиада научно-образовательного центра

«Эрудит»).

3. Составление групповых и индивидуальных проектов

по математике со студентами 1-2 курсов Минусинского

сельскохозяйственного колледжа.

Решение уравнений и неравенств c модулем

Версия для печати

1.4`;

1.3 Модуль неизвестного числа

`abs(x)=[({(x >= 0),(abs(x) = x):}), ({(x

1.4 Упражнения к определению модуля

1.4.1 Решить уравнение `abs(2x-1)=2x-1`
`2x-1>=0`; `x >= 1/2`
Ответ: `[1/2; +infty)`
1.4.2 Решить уравнение `abs(7x-14)=14-7x`
`7x-14
Ответ: `(-infty; 2]`
1.4.3 Решить уравнение `abs(3x-19)=1`
`[(3x-19=1), (3x-19=-1) :}`; `[(x=20/3), (x=6) :}`
Ответ: `{6; 20/3}`
1.4.4 Решить уравнение `abs(5x-11)=3x+4`
`{( 3x+4 >= 0 ), ( [(5x-11=3x+4), (5x-11=-3x-4):}) :}`; `{(x >= -4/3), ( [(x=15/2), (x=7/8):} ) :}`;
Ответ: `{7/8; 15/2}`
1.4.5 Решить неравенство `abs(x+4) <=1`

`-1
`-5
Ответ: `[-5; -3]`
1.4.2+2x-4=0) :}) :}`; `[( { (x >= 4/3), (x=2) :} ), ( { (x >= 4/3), ([(x=-1-sqrt5),(x=-1+sqrt5):}) :}) :}`;

Сравним `4/3 vv sqrt5-1` `4 vv 3sqrt5-3`; `7 vv 3sqrt5`; `49 vv 45 (>)`
`4/4 > sqrt5-1`
Ответ: 2

3. Решение неравенств, содержащих неизвестную функцию под знаком модуля

3.1 Смысл неравенства меньше либо равно


`abs(f(x)) <= varphi(x)`


Если `varphi(x)
Если `varphi(x) >= 0` — то необходимо решить систему: `{(varphi(x) >= 0), (-varphi(x)
`{(varphi(x) >= 0), (f(x) = -f(x)) :}`
Примеры с модулем на неравенства со смыслом меньше либо равно
3.1.1 Решить неравенство `abs(1-3x^2) <= 4x`

`{(x >= 0), (1-3x^2 <= 4x), (1-3x^2 >= -4x) :}`; `{(x >= 0), (3x^2+4x -1 >= 0), (3x^2 — 4x -1 <= 0) :}`

Ответ: `[(sqrt7-2)/3; (sqrt7+2)/3 ]`

3. 10`
`111 > 100` `9 < 10` `-37/20 < -5/3` `-3/2 > -5/3`

Итоговое объединение двух систем: `x < -5/3 uu [-3/2; 2] uu x>=2`

Ответ: `(-infty; -5/3) uu (-3/2; +infty)`

3.1.3 Найти область определения функции `f(x)=sqrt(1/2-abs(3/(5-x))`
`D_f(x)`:
`1/2-abs(3/(5-x)) >= 0`;
`abs(3/(5-x))
` -1/2

`{( 3/(5-x) <= 1/2), ( 3/(5-x) >= -1/2) :}`; `{( 3/(5-x) — 1/2 <= 0), ( 3/(5-x) + 1/2 >= 0):}`;

`{( (6-5+x)/(2(5-x)) <= 0 ), ( (6+5-x)/(2(5-x)) >= 0 ):}`; `2 > 0`; `{( (x +1)/(5-x) <= 0 ), ( (11-x)/(5-x) >= 0 ):}`;

Ответ: `(-infty; -1] uu [11; +infty)`

3.2/(x-1) <= 0) :} ) :}`; `[({(x <= 1/2), (x-1 > 0):}), ({(x <= 1/2), (-(x+1) <= 0):}) :}`; `[(emptyset), ({(x <= 1/2), (x+1 >= 0):}) :}`; `{(x <= 1/2), (x >= -1):}`

В ответ: `[-1; 1/2]` (б)
В ответ `[ ( a: (1/2; 1) uu (1; +infty)), ( б: [-1; 1/2]) :}`

Ответ: `[-1; 1) uu (1; +infty)`

4. Уравнения и неравенства, содержащие неизвестные под несколькими модулями

4.1 Решить неравенство `abs(x-4)-2abs(1-x) >= 1`

На числовом луче отметим значения x, при которых подмодульные значения обращаются в «0»: `x=1; x=4`. Луч разбился на три интервала.

Необходимо на каждом интервале найти решение данного неравенства, то есть решить совокупность трёх систем неравенств:
`[ ( { (x <= 1), (4-x-2(1-x) >= 1) :} ), ( {(1 < x <= 4), (4-x+2(1-x) >= 1) :} ), ( {(x > 4), (x-4+2(1-x) >= 1) :} ) :}` ; `[ ( { (x <= 1), (4-x-2+2x >= 1) :} ), ( {(1 < x <= 4), (4-x+2-2x >= 1) :} ), ( {(x > 4), (x-4+2-2x >= 1) :} ) :}` ;

`[ ( { (x <= 1), (x+1 >= 0) :} ), ( {(1 < x <= 4), (5-3x >= 0) :} ), ( {(x > 4), (-x-3 >= 0) :} ) :}` ; `[ ( { (x <= 1), (x >= -1) :} ), ( {(1 < x <= 4), (x <= 5/3) :} ), ( {(x > 4), (x <= -3) :} ) :}` ;


Ответ: `[-1; 5/3]`

4.2 -3abs(x)+1)`

Ответ: `(-infty; -5/3] uu {-1} uu {1} uu [5/3; +infty)`

6.9 Решить неравенство `3x — abs(x+8) — abs(1-x) <= -6`

Ответ: `(-infty; 1]`
Много задач с решениями на неравенства с модулем можно посмотреть здесь:
Решения неравенств с модулем

Алгебраическое и графическое решение уравнений, содержащих модули реферат по математике

Алгебраическое и графическое решение уравнений, содержащих модули Цель работы: хотя уравнения с модулями ученики начинают изучать уже с 6-го – 7-го класса, где они проходят самые азы уравнений с модулями. Я выбрал именно эту тему, потому что считаю, что она требует более глубокого и досканального исследования. Я хочу получить более широкие знания о модуле числа, различных способах решения уравнений, содержащих знак абсолютной величины. 1. Введение: Слово «модуль» произошло от латинского слова «modulus», что в переводе означает «мера». Это многозначное слово(омоним), которое имеет множество значений и применяется не только в математике, но и в архитектуре, физике, технике, програмировании и других точных науках. В архитектуре-это исходная еденица измерения, устанавливаемая для данного архитектурного сооружения и служащая для выражения кратных соотношений его составных элементов. В технике-это термин, применяемый в различных облостях техники, не имеющий универсального значения и служащий для обозначения различных коэффициентов и величин, например модуль зацепления, модуль упругости и .т.п. Модуль объемного сжатия( в физике)-отношение нормального напряжения в материале к относительному удлинению. 2. Понятия и определения Чтобы глубоко изучать данную тему, необходимо познакомиться с простейшими определениями, которые мне будут необходимы: Уравнение-это равенство, сродержащее переменные. Уравнение с модулем-это уравнение, содержащие переменную под знаком абсолютной величины(под знаком модуля).Например: |x|=1 Решить уравнение-это значит найти все его корни, или доказать, что корней нет. В математике модуль имеет несколько значений, но в моей исследовательской работе я возьму лишь одно: Модуль-абсолютная величина числа, равная расстоянию от начала отсчета до точки на числовой прямой. 3. Доказательство теорем Определение. Модуль числа a или абсолютная величина числа a равна a, если a больше или равно нулю и равна -a, если a меньше нуля: Из определения следует, что для любого действительного числа a, Теорема 1. Абсолютная величина действительного числа равна большему из двух чисел a или -a. Доказательство 1. Если число a положительно, то -a отрицательно, т. е. -a < 0 < a. Отсюда следует, что — a < a. Например, число 5 положительно, тогда -5 — отрицательно и -5 < 0 < 5, отсюда -5 < 5. В этом случае |a| = a, т. е. |a| совпадает с большим из двух чисел a и — a. 2. Если a отрицательно, тогда -a положительно и a < — a, т. е. большим числом является — a. По определению, в этом случае, |a| = -a — снова, равно большему из двух чисел -a и a. Следствие 1. Из теоремы следует, что |-a| = |a|. В самом деле, как , так и равны большему из чисел -a и a, а значит равны между собой. Следствие 2. Для любого действительного числа a справедливы неравенства Умножая второе равенство на -1 (при этом знак неравенства изменится на противоположный), мы получим следующие неравенства: справедливые для любого действительного числа a. Объединяя последние два неравенства в одно, получаем: Теорема 2. Абсолютная величина любого действительного числа a равна арифметическому квадратному корню из В самом деле, если то, по определению модуля числа, будем иметь С другой стороны, при значит |a| = Если a < 0, тогда |a| = -a и и в этом случае |a| = Эта теорема дает возможность при решении некоторых задач заменять |a| на Геометрически |a| означает расстояние на координатной прямой от точки, изображающей число a, до начала отсчета. Если то на координатной прямой существует две точки a и -a, равноудаленной от нуля, модули которых равны. Если a = 0, то на координатной прямой |a| изображается точкой 0 (см. рис.) Рис 4.Способы решения уравнений, содержащих модуль. Для решения уравнений, содержащих знак абсолютной величины, мы будем основыватся на определении модуля числа и свойствах абсолютной величины числа. Мы решим несколько примеров одним и тем же способом и посмотрим, какой из способов окажется проще для решения уравнений, содержащих модуль. Пример 1. Решитм аналитически и графически уравнение |x — 2| = 3. Решение Аналитическое решение 1-й способ Рассуждать будем, исходя из определения модуля. Если выражение, находящееся под модулем неотрицательно, т. е. x — 2 0, тогда оно «выйдет» из под знака модуля со знаком «плюс» и уравнение примет вид: x — 2 = 3. Если значения выражения под знаком модуля отрицательно, тогда, по определению, оно будет равно: или x — 2=-3 Таким образом, получаем, либо x — 2 = 3, либо x — 2 = -3. Решая полученные уравнения, находим: Рис. 11 Графики не пересекаются, значит уравнение не имеет решений (см. рис. 11). Ответ: нет решений. Пример 3. Решите аналитически и графически уравнение |-x + 2| = 2x + 1. Решение: Аналитическое решение 1-й способ Прежде следует установить область допустимых значений переменной. Возникает естественный вопрос, почему в предыдущих примерах не было необходимости делать этого, а сейчас она возникла. Дело в том, что в этом примере в левой части уравнения модуль некоторого выражения, а в правой части не число, а выражение с переменной, — именно это важное обстоятельство отличает данный пример от предыдущих. Поскольку в левой части — модуль, а в правой части, выражение, содержащее переменную, необходимо потребовать, чтобы это выражение было неотрицательным, т. е. Таким образом, область допустимых значений модуля Теперь можно рассуждать также, как и в примере 1, когда в правой части равенства находилось положительной число. Получим две смешанных системы: (1) и (2) Решим каждую систему: (1) входит в промежуток и является корнем уравнения. (2) x = -3 не входит в промежуток и не является корнем уравнения. Ответ: 2-й способ Установим, при каких значениях x модуль в левой части уравнения обращается в нуль: Получим два промежутка, на каждом из которых решим данное уравнение (см. рис. 12): Рис. 12 В результате будем иметь совокупность смешанных систем: Решая полученные системы, находим: (1) входит в промежуток и является корнем уравнения. (2) не входит в промежуток и x=-3 не является корнем уравнения Ответ: 4.1.Решение при помощи зависимостей между числами a и b, их модулями и квадратами этих чисел. Помимо приведенных мною выше способов существует определенная равносильность, между числами и модулями данных чисел, а также между квадратами и модулями данных чисел: |a|=|b| F 0D B a=b или a=-b a2=b2 F 0D B a=b или a=-b (1) Отсюда в свою очередь получим, что F 0 D B|a|=|b| a2=b2 (2) Пример 4. Решим уравнение |x + 1|=|2x – 5| двумя различными способами. 1.Учитывая соотношение (1), получим: x + 1=2x – 5 или x + 1=-2x + 5 x – 2x=-5 – 1 x + 2x=5 – 1 -x=-6|(:1) 3x=4 x=6 x=11/3 Корень первого уравнения x=6, корень второго уравнения x=11/3 Таким образом корни исходного уравнения x1=6, x2=11/3 2. В силу соотношения (2), получим (x + 1)2=(2x – 5)2, или x2 + 2x + 1=4×2 – 20x + 25 x2 – 4×2 +2x+1 + 20x – 25=0 -3×2 + 22x – 24=0|(:-1) 3×2 – 22x + 24=0 F 0 B 4 F 0 D ED/4=121-3 24=121 – 72=49>0 уравнение имеет 2 различных корня. x1=(11 – 7 )/3=11/3 x2=(11 + 7 )/3=6 Как показывает решение, корнями данного уравнения также являются числа 11/3 и 6 Ответ: x1=6, x2=11/3 Пример 5. Решим уравнение (2x + 3)2=(x – 1)2. Учитывая соотношение (2), получим, что |2x + 3|=|x – 1|, откуда по образцу предыдущего примера(и по соотношению (1)): 2х + 3=х – 1 или 2х + 3=-х + 1 2х – х=-1 – 3 2х+ х=1 – 3 х=-4 х=-0,(6) Таким образом корнями уравнения являются х1=-4, и х2=-0,(6) Ответ: х1=-4, х2=0,(6) Пример 6. Решим уравнение |x – 6|=|x2 – 5x + 9| Пользуясь соотношением (1), получим: х – 6=х2 – 5х + 9 или х – 6 = -(х2 – 5х + 9) -х2 + 5х + х – 6 – 9=0 |(-1) x – 6=-x2 + 5x — 9 x2 — 6x + 15=0 x2 – 4x + 3=0 F 0 2 A F 0 D E F 0 2 A F 0 D ED=36 – 4 15=36 – 60= -24 <0 D=16 – 4 3=4 >0 2 р.к. F 0 D E корней нет. x1=(4- 2 ) /2=1 x2=(4 + 2 ) /2=3 F 0 2 A F 0 2 AПроверка: |1 – 6|=|12 – 5 1 + 9| |3 – 6|=|32 – 5 3 + 9| 5 = 5(И) 3 = |9 – 15 + 9| 3 = 3(И) Ответ: x1=1; x2=3 4.2.Использование геометрической интерпритации модуля для решения уравнений. Геометрический смысл модуля разности величин-это расстояние между ними. Например, геометрический смысл выражения |x – a | -длина отрезка координатной оси, соединяющей точки с абсцисами а и х . Перевод алгеб-раической задачи на геометрический язык часто позволяет избежать громоздких решений. Пример7. Решим уравнение |x – 1| + |x – 2|=1 с использованием геометрической интерпритации модуля. Будем рассуждать следующим образом: исходя из геометрической интерпри-тации модуля, левая часть уравнения представляет собой сумму расстояний от некторой точки абсцисс х до двух фиксированных точек с абсциссами 1 и 2. Тогда очевидно, что все точки с абсциссами из отрезка [1; 2] обладают требуемым свойством, а точки, расположенные вне этого отрезка- нет. Отсюда ответ: множеством решений уравнения является отрезок [1; 2]. F 0 C EОтвет: х [1; 2] Пример8. Решим уравнение |x – 1| — |x – 2|=1 1 с использованием геометрической интерпритации модуля. Будем рассуждать аналогично предыдущему примеру, при этом получим, что разность расстояний до точек с абсциссами 1 и 2 равна единице только для точек, расположенных на координатной оси правее числа 2. Следовательно решением данного уравнения будет являтся не отрезок, заключенный между точками 1 и 2, а луч, выходящий из точки 2, и направленный в положительном направлении оси ОХ. F 0 C E F 0 A 5Ответ: х [2; + ) Обобщением вышеприведенных уравнений являются следующие равносильные переходы: |x – a| + |x – b|=b – a, где b F 0B 3 a F 0 D B a F 0 A 3 x F 0 A 3 b |x – a| — |x – b|=b – a, где b F 0B 3 a F 0 D B x F 0 B 3 b 4.3. Графики простейших функций, содержащих знак абсолютной величины Под простейшими функциями понимают алгебраическую сумму модулей линейных выражений. Сформулируем утверждение, позволяющее строить графики таких функций, не раскрывая модули ( что особенно важно, когда модулей достаточно много ): «Алгебраическая сумма модулей n линейных выражений представляет собой кусочно- линейную функцию, график которой состоит из n +1 прямолинейного отрезка. Тогда график может быть построен по n +2 точкам, n из которых представляют собой корни внутримодульных выражений, ещё одна — произвольная точка с абсциссой, меньшей меньшего из этих корней и последняя — с абсциссой, большей большего из корней. Например: 1)f(x)=|x — 1| Вычисляя функции в точках 1, 0 и 2, получаем график, состоящий из двух отрезков(рис.1) Уравнение равносильно системе Ответ: Пример12.Решить уравнение х2 — 4х +|x — 3| +3=0 Для освобождения от знака абсолютной величины разобьем числовую прямую на две области и будем искать решения исходного уравнения в каждой из этих областей отдельно: F 0 B 3__________x 3__________________|____________x<3_________________ |x – 3|=x – 3 |x – 3|=-x + 3 x2 — 4x + x – 3 + 3=0 x2 – 4x – x + 3 + 3=0 x2 – 3x=0 x2 – 5x + 6=0 x(x – 3) x1=0 или F 02 A F 0 D Ex2=3 D=25 – 4 6=1> 0 два различ. корня x=0 –посторонний корень, так как x1= (5- 1 )/2 =2 не удовлетворяет промежутку. x2=(5 + 1)/2=3 x=3 — посторонний корень, так как не удовлетворяет промежутку. Значит, исходное уравнение имеет два решения х1=2 и х2=3 Ответ: х1=2, х2=3 Пример13. Решить уравнение | 2x + 8 | – | x – 5 | = 12. Решение. Раскрытие пары модулей приводит к трем случаям (без F 0A 3 F 0 2 0 F 0 B 3x + 4 0, x – 5 0). Ответ: {– 25; 3}. Пример 14. Решить уравнение . Решение: Напишем равносильную смешанную систему: Ответ: х=-4 Пример 15 Решить графически уравнение |1 – x| — |2x + 3| + x + 4=0 Решение: Представим уравнение в виде |1 – x| — |2x + 3| =-х – 4 Построим два графика у=|1 – x| — |2x + 3| и у=-х – 4 1) у=|1 – x| — |2x + 3| Критические точки: х=1, х=-1.5 (1 – х) ________+________|______ +____________|_____-______ > (2х +3) — -1.5 + 1 + а) х< -1.5, (1– x)>0 и (2х + 3)<0, т.е функция примет вид у=1 – х + 2х + 3, у=х + 4 –графиком является прямая, проходящая через две точки (0; 4), (-4; 0) F 0 2 0 F 0 A 3б)При -1.5 x <1, (1 – х)>0 и (2x +3 F 0 B 3) 0, т.е функция примет вид у=1 – х – 2х -3, у=-3х – 2 –графиком является прямая, проходящая через две точки (0; -2), (-1; 1). F 0 2 0 F 0 B 3 F 0 2 0 F 0 A 3в)При х 1, (1 – х) 0 и (2х + 3)>0, т.е. функция примет вид у= -1 + х – 2х – 3, у= -х – 4 –графиком является прямая, проходящая через две точки (0; -4), (-4; 0). График функции у= — х – 4 совпадает с графиком у=|1 – x| — |2x + 3|, F 02 0 F 0 B 3при х 1, F 0 2 0 F 0 B 3Поэтому решением являются все х 1 и х= -4 F 0 2 0 F 0 B 3Ответ: х 1,х= -4 Аналитическое решение. y=|1 – x| — |2x + 3| y=-x – 4 Построим числовую прямую так, чтобы по определению модуля знак абсолютной величины числа можно будет снять. Для этого найдем критические точки: 1- х=0 и 2х – 3 =0, х=1 х=-1,5 F 0 A 3___________х<-1,5_____|_______-1,5 F 0 B 3x <1_____|_________x 1__________ |1 – x|=1 – x |1 – x|=1 – x |1 – x|=-1 + x |2x + 3|=-2x – 3 |2x + 3|=2x + 3 |2x + 3|=2x + 3 1 – x + 2x + 3 + x + 4=0 1 – x – 2x – 3 + x +4=0 -1 + x – 2x – 3 + x + 4=0 2x=-8 -2x=-2 0x=0 F 0 D Ex=-4 x=1 x – любое число. F 0 D EУдовлетворяет данному Не удовлетворяет F 0 C E F 0 A 5x [1; + ) F 0 D E F 0 D Eпромежутку является данному промежут- F 0 B 3x 1 корень уравнения F 0 D Eкорнем уравнения. ку не является кор- нем уравнения. Объеденив данные промежутки, получим, что решением данного уравнения являются: x=-4 и F 0B 3x 1 Ответ: F 0B 3x=-4, x 1

Решение уравнений с модулем — презентация онлайн

1. Решение уравнений с модулем

Учителя МОУ СОШ №23
Сурмалян Л.М.
Кущевский район.

2. Обучение- это ремесло, использующее бесчисленное количество маленьких трюков.

3. Задание 1. IХ-3I=5 Отметьте точки, координаты которых удовлетворяют указанному условию. Запишите их координаты.

Отметим точки, удаленные от точки А на 5 единичных
отрезков. Запишем их координаты.

4. Геометрическая интерпретация

Уравнение Iх – аI = b, где b > 0, допускает простую
геометрическую интерпретацию.
Решить уравнение Iх – 1I = 3 – значит найти все
точки числовой оси, которые отстоят от точки с
координатой (1) на расстоянии 3.
Ответ: -2 ; 4.

5. Геометрическая интерпретация

Решить уравнение Iх + 2I = 3 – значит найти все
точки числовой оси, которые отстоят от точки
с координатой (-2) на расстоянии 3.
Ответ: -5 ; 1.

6. Задание 2. Решите уравнения

IХ – 4I = 3
I х + 2 I= 7
ответ 1; 7
5; -9

7. Метод интервалов

Уравнение вида:
b1I x – a1I + … + bnI x – anI = b,
где a1
решается методом интервалов.
Суть: точки a1 , a2 , …, an числовую ось делят на
непересекающиеся промежутки знакопостоянства.
Решаем уравнение на каждом промежутке;
совокупность решений на всех промежутках и
составит решение исходного уравнения.

8. Решение уравнений.


x
-x+2-x-3=7
X=-4
Iх — 2I + Iх + 3I = 7

-3
-x+2+x+3=7
решений нет
Ответ: -4; 3.

x>2
x-2+x+3=7
x=3

9. Решение уравнений.


x
-x+5+x-2=3
X
Iх — 5I — Iх — 2I = 3

2
-x+5-x+2=3
x=2
Ответ: x

x>5
x-5-x+2=3
решений нет

10. Геометрическая интерпретация

Уравнения
Ix – aI + Ix – bI =c и Ix – aI — Ix – bI =c
имеют простую геометрическую интерпретацию.
Вернемся к предыдущим уравнениям.

11. Пример 1.

Решить уравнение
Iх — 2I + Iх + 3I = 7 – это
значит найти все точки на числовой оси Ох, для
каждой из которых сумма расстояний до точек с
координатами (2) и (-3) равна 7. Внутри отрезка
таких точек нет, так как длина меньше семи, значит
точки вне отрезка.
Ответ: -4; 3.

12. Пример 2.

Решить уравнение
Iх — 5I — Iх — 2I = 3 – это
значит найти все точки на числовой оси Ох, для
каждой из которых разность расстояний от нее до
точки с координатой (5) и расстояний от нее до
точки с координатой (2) равнялось 3. Длина отрезка
равна 3 следовательно любая точка левее (2) будет
решением уравнения.
Ответ: x

13. Пример 3.

IX — 1I + IX — 2I + IX — 3I =2
Построим графики функций:
Y=IX — 1I +
y
4
0
2
4
IX — 3I и Y= 2 — IX — 2I
пересечение графиков
точка (2;2)
Ответ: 2.
6
x

14. Обобщение.

Если в уравнении Ix – aI + Ix – bI =c,
Iа – bI
отрезка [a;b];
а если Ia – bI=c, то отрезок [a;b] будет
решением уравнения;
если Ia – bI>c, то уравнение решений иметь
не будет.

15. Обобщение.

Если в уравнении Ix – aI — Ix – bI =c,
Iа – bI = c, то при a b
a > b, x
если Ia – bI
если Ia – bI>c, то решение лежит внутри
отрезка [a;b].

16. Домашняя работа.

IX + 3I + IX — 3I =6
IX — 1I + IXI = 9
IX — 3I + IX — 1I =3
IX + 6I + IX + 4I =5
IX — 1I – IX + 1I = 3
I5 + XI – Iх – 8I = 13
IX — 3I + 2IX + 1I =4
IX — 4I + IX — 2I = IX+ 1I
IX+1I +IX-2I+IX-5I=6
Калькулятор по модулю

[Примеры модификаций]

Этот калькулятор по модулю — удобный инструмент, если вам нужно найти результат операций по модулю. Все, что вам нужно сделать, это ввести начальное число x и целое число y , чтобы найти число по модулю r , согласно x mod y = r . Читайте дальше, чтобы узнать, что такое операции по модулю, как вычислить по модулю и как правильно использовать этот калькулятор.

Что такое операции по модулю?

Представьте себе часы, висящие на стене.Допустим, уже поздно — 23 часа. Вы задаетесь вопросом, во сколько вы проснетесь после 8 часов сна. Вы не можете просто прибавить 8 к 11, потому что нет такого времени, как 19 часов утра. Чтобы найти правильный ответ, вам нужно выполнить операцию по модулю (mod 12) — вы складываете эти два числа и продолжаете вычитать 12, пока не получите число меньше 12. В этом случае 7. Вы только что подсчитали, что проснетесь в 7 утра.

Операции по модулю в случае часов настолько интуитивно понятны, что мы их даже не замечаем.В математике есть много типов более сложных операций по модулю, которые требуют большего осмысления. Мы можем записать это:

x mod y = r

истинно, если такое целое число q (так называемое частное ) существует, тогда:

y * q + r = x .

В противном случае число r — это остаток от деления, где x — это дивиденд , а y делитель .

Если определение по модулю вам не нравится, и вы все еще не знаете, как вычислить по модулю, посмотрите следующий абзац, и все должно стать кристально ясным.

Что такое сравнение по модулю?

Два числа a и b считаются равными по модулю n , когда их разность a - b целиком делится на n (так что (a - b) кратно n ).

Математически формула сравнения по модулю записывается как:

a ≡ b (мод. N)

и n называется модулем конгруэнции.

С другой стороны, вы можете сказать, что a и b считаются равными по модулю n , когда они оба имеют одинаковый остаток при делении на n:

мод n = r

b мод n = r

, где r — общий остаток.

Итак, проще говоря — совпадение по модулю происходит, когда два числа имеют одинаковый остаток после одного и того же делителя, например: 24 по модулю 10 и 34 по модулю 10 дают тот же ответ: 4.Следовательно, 24 и 34 сравнимы по модулю 10.

Давайте посмотрим на другой пример:

9 ≡ 21 (мод.6) ,

, потому что 21 - 9 = 12 делится на 6. Его также можно записать коротко как 6 | (21 - 9) . Или, что то же самое, 21 и 9 имеют одинаковый остаток, когда мы делим их на 6:

9 мод 6 = 3

21 mod 6 = 3

Как вычислить по модулю — пример

Рассчитать модуль вручную — несложная задача.Просто следуйте инструкциям ниже!

  1. Начните с выбора начального числа (перед выполнением операции по модулю). Допустим, 250. Это наши дивиденды.
  2. Выберите делитель. Возьмем 24. Операция, которую мы хотим вычислить, тогда будет 250 mod 24 ( 250% 24 , если используется другое соглашение).
  3. Разделите одно число на другое с округлением в меньшую сторону: 250/24 = 10 . Это частное. Кроме того, вы можете думать об этой операции как о делении целого числа на — типа деления, при котором нам не важна дробная часть результата.
  4. Умножьте делитель на частное. Итак, в нашем примере это 10 * 24 = 240 .
  5. Вычтите это число из вашего начального числа (делимого). Здесь: 250 - 240 = 10 .
  6. Полученное число является результатом операции по модулю. Мы можем записать это как 250 mod 24 = 10 .

Как пользоваться нашим калькулятором модов? 10 mod 3 и другие примеры по модулю

Определить модуль с помощью нашего инструмента просто и удобно.Чтобы найти результат операций по модулю между целыми числами, вам необходимо:

  1. Введите начальное число — делимое — в первое поле . Возьмем пример из предыдущего абзаца, поэтому введите 250.
  2. Введите делитель . В нашем случае 24.
  3. Тадааа! Наш калькулятор по модулю вернет вам результат — остаток! И это неудивительно, оно равно 10 — столько же, сколько мы рассчитывали ранее.

Ниже вы найдете несколько типичных запросов, касающихся модуля:

  • 1 mod 1 = 0 (поскольку mod 1 всегда равен 0)
  • 1 мод 2 = 1
  • 1 мод 3 = 1
  • 5 мод 2 = 1
  • 5 мод 3 = 2
  • 6 мод 3 = 0
  • 7 мод 3 = 1
  • 10 мод 3 = 1
  • 18 мод 3 = 0
  • 100 мод 3 = 1
  • 100 мод 7 = 2

Если вы не видите здесь тот, который хотите найти, воспользуйтесь нашим калькулятором по модулю!

Модульная арифметика

Модульная арифметика — это, вообще говоря, арифметическая система для целых чисел, в которой числа «оборачивают» определенное число.Подведем итог тому, что мы узнали о различных представлениях операций по модулю — все приведенные ниже утверждения являются эквивалентами:

  • A ≡ B (мод C)
  • А мод. C = B мод. C
  • C | (А - В)
  • A = B + K * C , где K — некоторое целое число

Мы также можем выполнять вычисления по модулю операций.

1. Модульное сложение и вычитание

(A + B) мод C = (A мод C + B мод C) мод C

(A - B) мод C = (A мод C - B мод C) мод C

Итак, сумма по модулю суммы двух чисел равна сумме по модулю этих чисел, вычисленных отдельно, а затем умноженной на делитель по модулю.Первый этап делается для того, чтобы избавиться от частной части, а затем снова используется операция mod. Взгляните на пример:

  • А = 11, В = 7, С = 4

    (11 + 7) по модулю 4 = (11 по модулю 4 + 7 по модулю 4) по модулю 4

    левая часть уравнения: (11 + 7) mod 4 = 18 mod 4 = 2

    правая часть уравнения: (11 mod 4 + 7 mod 4) mod 4 = (3 + 3) mod 4 = 6 mod 4 = 2

Аналогично, вычисления аналогичны для вычитания.

2. Модульное умножение

(A * B) мод C = (A мод C * B мод C) мод C

Такое уравнение может быть полезно при работе с большими числами, и мы не можем сразу узнать модуль этого большого числа. Давайте посмотрим на тот же пример (A = 11, B = 7, C = 4) — можете ли вы найти результат 77 mod 4 на месте? 11 mod 4 и 7 mod 4 вычислить проще:

  • (11 * 7) по модулю 4 = (11 по модулю 4 * 7 по модулю 4) по модулю 4

    левая часть уравнения: (11 * 7) mod 4 = 77 mod 4 = 1

    правая часть уравнения: (11 mod 4 * 7 mod 4) mod 4 = (3 * 3) mod 4 = 9 mod 4 = 1

3.100 мод 3 = (1 * 1) мод 3 = 1

Для некоторых конкретных случаев существуют даже более быстрые методы модульного возведения в степень (если B — степень двойки). Если вы хотите прочитать о них и попрактиковаться в модульной арифметике, ознакомьтесь с отличным учебником от Khan Academy под названием «Что такое модульная арифметика?»

Неопределенность определения модуля

Слово modulo происходит от латинского слова modus , означающего меру. Обычно, когда мы используем слово по модулю , мы имеем в виду операцию по модулю , например, e.г. 11 по модулю 3 равно 2, поэтому нужно просто найти остаток. В строгом смысле, модуль означает:

По указанному модулю

или

A то же самое, что B по модулю C, за исключением различий, учтенных или объясненных C

Это определение, о котором мы писали в сравнении по модулю абзаца.

Однако, по модулю используется не только в математическом контексте.Иногда вы можете услышать это в повседневном разговоре, где это, вероятно, означает игнорирование, неучет чего-то, с должным учетом чего-то, например:

Дизайн был лучшим до сих пор, по модулю частей, которые все еще нуждаются в доработке.

Percent — символ операции по модулю

Операция по модулю часто используется в языках программирования. Для этого% — процент — используется для обозначения этой операции (или иногда оператор остатка для отрицательных чисел).Если вам интересно узнать о происхождении знака%, мы настоятельно рекомендуем вам прочитать небольшой абзац, который мы составили об истории знака процента.

Вам нужно быть осторожным, так как при учете отрицательных значений есть некоторая двусмысленность с определением по модулю. Для остатка есть два возможных варианта — отрицательный и положительный, и результат зависит от реализации на выбранном языке программирования.

Приложения по модулю

На первый взгляд они могут быть неочевидными, но существует множество применений модуло — от повседневной жизни до задач по математике и естествознанию!

  1. Наиболее очевидным и известным примером является так называемая арифметика часов 🕞.Это может быть добавление часов, как в объяснении по модулю выше, или минут, или секунд! Никто не скажет, что «у вас осталось 40 минут 90 секунд », верно? Единственный вариант — выполнить операцию по модулю и найти частное и остаток — 60 * 1 + 30 = 90 . 41 минута 30 секунд звучит намного лучше.

  2. Операции по модулю используются для вычисления контрольных сумм серийных номеров. Контрольные цифры используются в основном в длинных числах, и это цифры, вычисляемые алгоритмом.Они готовы сообщить вам о возникающих ошибках, например от опечаток. Вы можете найти применение по модулю в:

  • Контрольные цифры GTIN, UPC, EAN используются для подтверждения целостности штрих-кода. В формуле для контрольных цифр используется модуль 10.
  • Номера
  • ISBN и ISSN , которые являются уникальными периодическими идентификаторами и идентификаторами книг, имеют модуль 11 или 10, а в формуле контрольной цифры применяется средний вес.
  • IBAN — Номера международных банковских счетов — используйте модуль 97, чтобы проверить, правильно ли клиент ввел номер.
  • NPI — Национальный идентификатор провайдера США использует операцию по модулю 10 для вычисления десятой цифры.

Поскольку контрольные цифры используются для выявления человеческих ошибок транскрипции, они часто используются для длинных серийных номеров. Другие примеры алгоритмов контрольных цифр с использованием операций по модулю:

  • национальный идентификационный номер (например, в Исландии, Турции, Польше)
  • фискальный идентификационный номер (Испания)
  • идентификационный номер автомобиля (США)
  • и многие, многие другие.
  1. Он применяется во многих научных областях, таких как компьютерная алгебра, криптография, информатика или простая школьная математика — как в алгоритме Евклида для вычисления наибольшего общего множителя.

  2. Modulo полезен, когда вам нужно что-то разделить. Примером из реальной жизни может быть разделение пиццы с друзьями или семьей.

Предположим, что в большой пицце для вечеринки 10 ломтиков, а вы — группа из трех человек.Сколько кусочков останется, если пиццу разделить поровну?

Это именно тот случай, когда вы можете использовать по модулю! 10 mod 3 = 1. Другими словами, 10, разделенное на 3, равняется 3, но остается 1 кусок 🍕. Это был не самый сложный пример, но мы надеемся, что вы видите полезность модуло.

Кстати , а вы видели нашу коллекцию калькуляторов пиццы? У нас есть удивительный калькулятор вечеринки с пиццей, который может помочь оценить, сколько пиццы вам нужно заказать, а также инструменты, помогающие сравнить размеры пиццы — если вы когда-нибудь задумывались, что лучше купить две пиццы среднего размера или одну большую, пиццу Калькулятор сравнения — беспроигрышный вариант.n = \ text {произведение нескольких выражений}, получается = произведение нескольких выражений. В частности, если aaa простое число, то по основной арифметической теореме мы можем заключить, что каждый множитель в произведении является степенью этого простого числа. Даже если aaa является составным, он часто сводится к рассмотрению случая после рассмотрения наибольшего общего делителя. Чтобы проиллюстрировать эту технику, мы приводим следующий пример:

Решите относительно (p, n, x) (p, n, x) (p, n, x) в следующем диофантовом уравнении с простым ppp и положительными целыми числами x, n: x, n: x, n:

пн + 1 = х2.z.3x + 4y = 5z.

Подсказка: покажите, что x = y = 2x = y = 2x = y = 2 — единственные решения.

Развлечение с модульной арифметикой — лучшее объяснение

Читатель недавно предложил мне написать о модульной арифметике (также известной как «взятие остатка»). Я не особо задумывался над этим, но понял, что модуль по модулю чрезвычайно эффективен: он должен быть в нашем ментальном наборе инструментов рядом со сложением и умножением.

Вместо того, чтобы бить вас по лицу формулами, давайте исследуем идею, которой мы незаметно подвергались в течение многих лет.Есть хорошая статья о модульной арифметике, которая вдохновила этот пост.

Нечетный, четный и тройной

Вскоре после обнаружения целых чисел (1, 2, 3, 4, 5…) мы поняли, что они делятся на две группы:

  • Четное: делится на 2 (0, 2, 4, 6 ..)
  • Нечетное: не делится на 2 (1, 3, 5, 7…)

Почему это различие важно? Это начало абстракции — мы замечаем свойства числа (например, четное или нечетное), а не только само число («37»).

Это огромно — это позволяет нам исследовать математику на более глубоком уровне и находить взаимосвязи между типами и числами, а не конкретными. Например, мы можем составить такие правила:

  • Четный x Четный = Четный
  • Нечетный x Нечетный = Нечетный
  • Четный x Нечетный = Четный

Эти правила общие — они действуют на уровне собственности. (Интуитивно у меня есть химическая аналогия, что «ровность» — это молекула, которую имеют некоторые числа, и ее нельзя удалить умножением.)

Но четный / нечетный — это очень специфическое свойство: деление на 2. А как насчет числа 3? Как насчет этого:

  • «Три четверти» означает, что число делится на 3 (0, 3, 6, 9…)
  • «Throdd» означает, что вы , а не , делимое на 3 (1, 2, 4, 5, 7, 8…)

Странно, но работоспособно. Вы заметите несколько вещей: есть два типа тредда. Число вроде «4» находится на расстоянии 1 от тройки (остаток 1), а число 5 — от двух (остаток 2).

Быть «тройкой» — это еще одно свойство числа.Возможно, не так быстро, как четный / нечетный, но он есть: мы можем создавать правила вроде «три семь х три четверти = три четверти» и так далее.

Но это сходит с ума. Мы не можем постоянно придумывать новые слова.

Введите модуль

Операция по модулю (сокращенно «mod» или «%» на многих языках программирования) является остатком при делении. Например, «5 mod 3 = 2» означает, что 2 — это остаток от деления 5 на 3.

Преобразуя повседневные термины в математические, «четное число» — это такое, где оно равно «0 по модулю 2», то есть имеет остаток 0 при делении на 2.Нечетное число — «1 mod 2» (остаток равен 1).

Почему это круто? Итак, наши «нечетные / четные» правила становятся такими:

  • Четный x Четный = 0 x 0 = 0 [четный]
  • Нечетный x Нечетный = 1 x 1 = 1 [нечетный]
  • Четный x Нечетный = 0 x 1 = 0 [Четный]

Круто, а? Довольно легко решить — мы преобразовали «свойства» в реальные уравнения и обнаружили некоторые новые факты.

Что такое четный x четный x нечетный x нечетный? Ну, это 0 x 0 x 1 x 1 = 0. На самом деле, вы можете увидеть, если есть четное умножение где-нибудь на , весь результат будет нулем … я имею в виду даже :).

Математические часы

Подлость в модульной математике заключается в том, что мы уже использовали ее для отсчета времени — иногда называемой «арифметикой часов».

Например: 7:00 (am / pm не имеет значения). Где будет часовая стрелка через 7 часов?

грн. 7 + 7 = 14, но мы не можем показать «14:00» на часах. Значит, должно быть 2. Мы рассуждаем интуитивно и математически:

  • (7 + 7) mod 12 = (14) mod 12 = 2 mod 12 [2 — остаток от деления 14 на 12]

Уравнение «14 по модулю 12 = 2 по модулю 12» означает, что «14 часов» и «2 часа» выглядят одинаково в 12-часовых часах.Это конгруэнтных , обозначенных тройным знаком равенства: 14 ≡ 2 mod 12.

Другой пример: сейчас 8:00. Где будет большая рука через 25 часов?

Вместо того, чтобы прибавлять 25 к 8, вы можете понять, что 25 часов — это просто «1 день + 1 час». Итак, часы закончатся на 1 час вперед, в 9:00.

  • (8 + 25) по модулю 12 (8) по модулю 12 + (25) по модулю 12 ≡ (8) по модулю 12 + (1) по модулю 12 ≡ 9 по модулю 12

Вы интуитивно преобразовали 25 в 1 и добавили это к 8.

Интересное свойство: математика просто работает

Используя аналогию с часами, мы можем выяснить, работают ли правила модульной арифметики (они работают).

Сложение / вычитание

Допустим, два раза на наших часах выглядят одинаково («2:00» и «14:00»). Если мы добавим к обоим одинаковые «x» часов, что произойдет?

Ну меняют на столько же на часах! 2:00 + 5 часов ≡ 14:00 + 5 часов — оба покажут 7:00.

Почему? Ну, нас никогда не волновали лишние «12:00», которые таскала с собой 14. Мы можем просто добавить 5 к двум остаткам, которые есть у обоих, и они продвинутся одинаково. Для всех совпадающих чисел (2 и 14) сложение и вычитание имеют одинаковый результат.

Умножение

Труднее увидеть, останется ли умножение прежним. Если 14 ≡ 2 (mod 12), можем ли мы умножить обе части и получить тот же результат?

Давайте посмотрим — что произойдет, если мы умножим на 3?

Ну, 2:00 * 3 ≡ 6:00. Но что такое «14:00» * 3?

Помните, 14 = 12 + 2. Итак, мы можем сказать

  • 14 * 3 = (12 + 2) * 3 = (12 * 3) + (2 * 3) mod 12

Первую часть (12 * 3) можно игнорировать! «12-часовое переполнение», которое несет 14, просто повторяется несколько раз.Но кого это волнует? В любом случае мы игнорируем переполнение.

При умножении имеет значение только остаток, который равен 2 часам для 14:00 и 2:00. Интуитивно вот как я вижу, что умножение не меняет отношения с модульной математикой (вы можете перемножить обе стороны модульного отношения и получить тот же результат). См. Ссылку выше для более строгих доказательств — это мои интуитивно понятные карандашные линии.

Использование модульной арифметики

А теперь самое интересное — чем полезна модульная арифметика?

Простые расчеты времени

Мы делаем это интуитивно, но неплохо дать этому название.Ваш рейс прибывает в 15:00. Задерживается на 14 часов. Во сколько он приземлится?

Ну, 14 2 mod 12. Я думаю об этом как о «2 часа и переключение между утра и вечера», поэтому я знаю, что это будет «3 + 2 = 5 утра».

Это немного сложнее, чем простой оператор по модулю, но принцип тот же.

Размещение элементов в случайных группах

Предположим, у вас есть люди, которые купили билеты в кино с номером подтверждения. Вы хотите разделить их на 2 группы.

Чем вы занимаетесь? «Здесь шансы, вон там». Вам не нужно знать, сколько билетов было выписано (первая половина, вторая половина), каждый может мгновенно определить свою группу (без обращения в центральный орган), и схема работает по мере того, как все больше людей покупают билеты.

Нужны 3 группы? Разделите на 3 и возьмите остаток (также известный как мод 3). У вас будут группы «0», «1» и «2».

В программировании взятие по модулю — это то, как вы можете поместить элементы в хеш-таблицу: если ваша таблица имеет N записей, преобразуйте ключ элемента в число, выполните модификацию N и поместите элемент в это ведро (возможно, сохраняя связанный список там).По мере увеличения размера вашей хеш-таблицы вы можете пересчитать модуль для ключей.

Выбор случайного предмета

Я использую модуло в реальной жизни. Действительно. У нас есть 4 человека, играющих в игру, и нам нужно выбрать кого-нибудь, кто будет первым. Сыграйте в мини-игру с модом N! Дайте людям номера 0, 1, 2 и 3.

Теперь все говорят: «Раз, два, три, стреляйте!» и выставляет случайное количество пальцев. Сложите их и разделите на 4 — тот, кто получит остаток, ходит первым.(Например: если сумма пальцев равна 11, тот, у кого было «3», ходит первым, так как 11 mod 4 = 3).

Это быстро и работает.

Выполнение задач в цикле

Предположим, что задачи должны выполняться по определенному расписанию:

  • Задача A выполняется 3 раза в час
  • Задача B выполняется 6 раз в час
  • Задача C выполняется 1 раз в час

Как сохранить эту информацию и составить расписание? В одну сторону:

  • Установить таймер, работающий каждую минуту (значение минут обозначается буквой «n»)
  • 3x / час означает каждые 60/3 = 20 минут.Таким образом, задача A запускается всякий раз, когда «n% 20 == 0»
  • Задача B запускается всякий раз, когда «n% 10 == 0»
  • Задача C запускается всякий раз, когда «n% 60 == 0»

О, вам нужна задача C1, которая выполняется 1 раз в час, но не в то же время, что и задача C? Конечно, пусть он запускается, когда «n mod 60 == 1» (все еще один раз в час, но не то же самое, что и C1).

Мысленно я вижу цикл, который я хочу «поразить» через различные промежутки времени, поэтому вставляю мод. Приятно то, что попадания могут перекрываться независимо друг от друга. В этом отношении это немного похоже на XOR (каждый XOR может быть многоуровневым, но это уже другая статья!).

Аналогично, при программировании вы можете распечатать каждый сотый элемент журнала, выполнив: if (n% 100 == 0) {print…}.

Это очень гибкий и простой способ запускать элементы по расписанию. Фактически, это ответ на проверку работоспособности FizzBuzz. Если в вашем поясе нет операции по модулю, вопрос становится намного сложнее.

Поиск свойств номеров

Предположим, я сказал вам это:

Что можно быстро сделать? Что ж, «а» должно быть четным, поскольку оно равно чему-то, что подразумевает умножение на 2.

Если бы я также сказал вам:

Вы бы не отказались. Не потому, что вы «знаете», что эти два продукта разные, а потому, что один явно четный, а другой — нечетный. Возникла проблема: в обоих случаях a не может быть одним и тем же числом, поскольку свойства не совпадают с .

Такие вещи, как «even», «threeven» и «mod n» являются свойствами, которые являются более общими, чем отдельные числа, и которые мы можем проверить на согласованность. Таким образом, мы можем использовать по модулю, чтобы выяснить, согласуются ли числа, не зная, что это такое!

Если я скажу вам это:

Можно ли решить эти уравнения с целыми числами? Посмотрим:

  • 3a + 5b = 8… давайте «mod 3 it»: 0 + 2b ≡ 2 mod 3 или b ≡ 1 mod 3
  • 3a + b = 2… давайте «mod 3 it»: 0 + b ≡ 2 mod 3), или b ≡ 2 mod 3

Противоречие, молодцы! B не может быть одновременно «1 mod 3» и «2 mod 3» — это так же абсурдно, как быть четным и нечетным одновременно!

Но есть одна проблема: числа вроде «1.5 ”не являются ни четными, ни нечетными — они не целые числа! Модульные свойства применяются к целым числам, поэтому мы можем сказать, что b не может быть целым числом .

Потому что на самом деле мы можем решить это уравнение:

  • (3a + 5b) — (3a + b) = 8-2
  • 4b = 6
  • б = 1,5
  • 3a + 1,5 = 2, поэтому 3a = 0,5 и a = 1/6

Не соблазняйтесь силой по модулю! Знайте его пределы: это применимо к целым числам.

Криптография

Игра с числами имеет очень важное применение в криптографии.Здесь слишком много, чтобы рассказывать о нем, но модуль по модулю используется в обмене ключами Диффи-Хеллмана — используется при настройке SSL-соединений для шифрования веб-трафика.

Обычный английский

Гики любят использовать технические слова в обычном контексте. Вы можете услышать «X совпадает с Y по модулю Z», что примерно означает «игнорирование Z, X и Y одинаковы».

Например:

  • b и B идентичны, по модулю капитализации
  • iTouch и iPad идентичны, по модулю размера;)

Вперед и вверх

Странно думать о «полезности» оператора по модулю — это как если бы кто-то спрашивал, почему полезны показатели.В повседневной жизни не очень, но это инструмент, позволяющий понять закономерности в мире и создать свои собственные.

В общем, я вижу несколько общих вариантов использования:

  • Редуктор диапазона: возьмите вход, мод N, и у вас есть число от 0 до N-1.
  • Назначение группы: возьмите вход, мод N, и вы пометите его как группу от 0 до N-1. Эта группа может быть согласована любым количеством сторон — например, разные серверы, которые знают N = 20, могут согласовать, к какой группе принадлежит ID = 57.
  • Вычислитель свойств: обрабатывайте числа в соответствии с их свойствами (четные, тройные и т. Д.) И вырабатывайте принципы, полученные на уровне свойств

Я уверен, что я упустил еще несколько десятков применений — не стесняйтесь комментировать ниже.Удачной математики!

Другие сообщения этой серии

  1. Методы сложения чисел от 1 до 100
  2. Переосмысление арифметики: наглядное руководство
  3. Quick Insight: интуитивное значение подразделения
  4. Quick Insight: вычитание отрицательных чисел
  5. Удивительные узоры в квадратных числах (1, 4, 9, 16…)
  6. Развлечение с модульной арифметикой
  7. Учимся считать (как избежать проблемы со столбами забора)
  8. Причудливое введение в системы счисления
  9. Другой взгляд на простые числа
  10. Интуиция к золотому сечению
  11. Различные интерпретации числа ноль

Линейные уравнения ADS по модулю целых чисел 2

Подраздел 12.6.1 Уменьшение ряда, мод 2

Методы, которые мы изучили для решения систем уравнений до этого момента, могут быть применены к системам, в которых вся арифметика выполняется над другими алгебраическими системами, включая целые числа по модулю 2. Случай mod 2 станет особенно полезным в нашем дальнейшем исследовании теория кодирования.

При решении систем уравнений с арифметикой по модулю 2 элементарные операции со строками по-прежнему остаются фундаментальными. Однако, поскольку существует только один ненулевой элемент, 1, вам никогда не нужно умножать строку на ненулевую константу.{n-m} \) разные решения.

Рассмотрим пример, который сразу же переводится в матричную форму.

\ begin {уравнение *} \ begin {array} {r @ {} r @ {} r @ {} r @ {} r @ {} r @ {} r @ {} r @ {} r @ {} r @ {} r @ {} р@{}} x_1 & {} + {} & x_2 & & & {} + {} & x_4 & & & & & = 1 \\ x_1 & & & {} + {} & x_3 & & & {} + {} & x_5 & & & = 0 \\ & & x_2 & {} + {} & x_3 & & & & & {} + {} & x_6 & = 0 \\ \ end {массив} \ end {уравнение *}

Расширенная матрица системы

\ begin {уравнение *} \левый( \ begin {array} {cccccc | c} 1 & 1 & 0 & 1 & 0 & 0 & 1 \\ 1 & 0 & 1 & 0 & 1 & 0 & 0 \\ 0 & 1 & 1 & 0 & 0 & 1 & 0 \\ \ end {массив} \верно) \ end {уравнение *}

Шаги по уменьшению строк этой матрицы приведены ниже.Записи, по которым мы «делаем поворот», выделены жирным шрифтом, чтобы было легче идентифицировать основные переменные.

\ begin {уравнение *} \ begin {split} \левый( \ begin {array} {cccccc | c} 1 & 1 & 0 & 1 & 0 & 0 & 1 \\ 1 & 0 & 1 & 0 & 1 & 0 & 0 \\ 0 & 1 & 1 & 0 & 0 & 1 & 0 \\ \ end {массив} \ right) & \ overset {\ textrm {add} R_1 \ textrm {to} R_2} {\ text {} \ longrightarrow} \ text {} \левый( \ begin {array} {cccccc | c} \ textbf {1} & 1 & 0 & 1 & 0 & 0 & 1 \\ 0 & 1 & 1 & 1 & 1 & 0 & 1 \\ 0 & 1 & 1 & 0 & 0 & 1 & 0 \\ \ end {массив} \верно)\\ & \ text {} \ overset {\ textrm {add} R_2 \ textrm {to} R_1} {\ text {} \ longrightarrow} \ text {} \ left ( \ begin {array} {cccccc | c} \ textbf {1} & 0 & 1 & 0 & 1 & 0 & 0 \\ 0 & \ textbf {1} & 1 & 1 & 1 & 0 & 1 \\ 0 & 1 & 1 & 0 & 0 & 1 & 0 \\ \ end {массив} \верно) \\ & \ overset {\ textrm {add} R_2 \ textrm {to} R_3} {\ text {} \ longrightarrow} \ text {} \левый( \ begin {array} {cccccc | c} \ textbf {1} & 0 & 1 & 0 & 1 & 0 & 0 \\ 0 & \ textbf {1} & 1 & 1 & 1 & 0 & 1 \\ 0 & 0 & 0 & 1 & 1 & 1 & 1 \\ \ end {массив} \верно) \ end {split} \ end {уравнение *}

Обратите внимание, что в этот момент мы не можем повернуться к третьей строке, третьему столбцу, поскольку эта запись равна нулю.Поэтому мы переходим к следующему столбцу, делая \ (x_4 \) основным.

\ begin {уравнение *} \ begin {split} \ text {} & \ overset {\ textrm {add} R_3 \ textrm {to} R_2} {\ text {} \ longrightarrow} \ text {} \левый( \ begin {array} {cccccc | c} \ textbf {1} & 0 & 1 & 0 & 1 & 0 & 0 \\ 0 & \ textbf {1} & 1 & 0 & 0 & 1 & 0 \\ 0 & 0 & 0 & \ textbf {1} & 1 & 1 & 1 \\ \ end {массив} \верно) \ end {split} \ end {уравнение *}

На этом сокращение строки завершено, и теперь мы можем идентифицировать набор решений.Имейте в виду, что, поскольку сложение — это вычитание, члены можно перемещать в любую сторону от знака равенства без изменения знака. Основными переменными являются \ (x_1 \ text {,} \) \ (x_2 \ text {,} \) и \ (x_4 \ text {,} \), а остальные три переменные свободны. Общее решение системы

\ begin {уравнение *} \ begin {array} {c} х_1 = х_3 + х_5 \\ х_2 = х_3 + х_6 \\ х_3 = х_3 \\ х_4 = 1+ х_5 + х_6 \\ х_5 = х_5 \\ х_6 = х_6 \\ \ end {массив} \ end {уравнение *}

С тремя свободными переменными существует \ (2 ^ 3 = 8 \) решений этой системы.Например, один из них получается установкой \ (x_3 = 1 \ text {,} \) \ (x_5 = 1 \ text {,} \) и \ (x_6 = 0 \ text {,} \), что дает \ ((x_1, x_2, x_3, x_4, x_5, x_6) = (0,1,1,0,1,0) \ text {.} \)

Мы можем проверить сокращение строк с помощью SageMath:

Евклидов алгоритм и решение конгруэнций

Джозеф Малькевич: евклидов алгоритм и решение конгруэнций

Евклидов алгоритм и решающие сравнения

Подготовлено:

Джозеф Малькевич
Департамент математики и компьютерных исследований
Йоркский колледж (CUNY)
Ямайка, Нью-Йорк 11451

Если d — наибольший общий делитель двух (положительный) целые числа a и b, затем d разделят остаток r, полученный в результате деления меньшего из чисел a и b на большее.

Пример 1:

Наибольший общий делитель 24 и 16 равен 8.

24 = 1 (16) + 8. Следовательно, остаток от деления 24 на 16 равен 8, что делится на 8.

Пример 2:

Наибольший общий делитель 100 и 60 равен 20.

100 = 1 (60) + 40. Остаток 40 делится на 20.

Приведенный выше факт составляет основу того, что известно как Евклидов алгоритм нахождения наибольшего общего делителя двух чисел. Алгоритм назван в честь того же математика Евклида, который известен своими работами в области геометрии.Прелесть этого алгоритма в том, что он не находит наибольший общий делитель, используя факторизацию на простые числа. Факторизация больших чисел представляется очень сложной проблемой. В настоящее время неизвестно, какова точная вычислительная сложность факторизации, но это один из наиболее широко используемых алгоритмов криптографии (например, RSA, названный в честь компьютерных ученых Рональда Ривеста, Ади Шамира и Леонарда Адельмана, которые разработали метод, который был опубликован в 1978 году. в то время как они были студентами Массачусетского технологического института) зависит от его безопасности при предположении, что факторинг больших чисел очень сложно.

Для заданных a и b разделите меньшее и большее на большее, получив частное q и остаток r.

Предполагая, что b больше a, мы имеем:

b = q (a) + r. Теперь наибольший общий делитель b и a совпадает с наибольшим общим делителем a и r.

Повторяя процесс для a и r (вместо a и b), мы в конечном итоге приходим к ситуации, когда остаток равен нулю. Это свойство алгоритма деления, что полученный остаток r должен быть меньше числа b.Наибольший общий делитель будет последним ненулевым остатком в этом процессе.

Пример 3:

Найдите наибольший общий делитель 36 и 10.

36 = 3 (10) + 6

10 = 1 (6) + 4

6 = 1 (4) + 2

4 = 2 (2) + 0

Последний ненулевой остаток равен 2, и, следовательно, наибольший общий делитель 10 и 36 равен 2.

Теорема 1:

Если d является наибольшим общим делителем a и b, то d можно записать в виде:

d = ax + по (1)


где x и y целые числа.

В частности, если два числа являются относительно простыми , то есть их наибольший общий делитель равен 1, то мы можем заключить, что:

для некоторых целых чисел x и y.

Давайте найдем значения x и y для примера 3. В этом примере мы увидели, что наибольший общий делитель 36 и 10 равен 2.

Мы увидели, что:

36 = 3 (10) + 6 ( 2)

10 = 1 (6) + 4 (3)

6 = 1 (4) + 2 (4)

4 = 2 (2) + 0

Из уравнения (4) видно, что Наибольший общий делитель 36 и 10 равен 2, и что:

2 = (1) (6) -1 (4) (5)

Но мы знаем из уравнения (3), что 4 = 1 (10) — 1 ( 6), поэтому мы можем заменить значение 4 в уравнении (5) на 1 (10) — 1 (6), чтобы получить:

2 = (1) (6) — 1 (1 (10) — 1 (6) ) = (1) (6) -1 (10) + 1 (6) = 2 (6) — 1 (10).(6)

Но мы знаем из уравнения (2), что 6 = 1 (36) — 3 (10), поэтому мы можем заменить значение 6 в уравнении (6) на 1 (36) — 3 (10), чтобы получить :

2 = 2 (1 (36) — 3 (10)) -1 (10) = 2 (36) — 6 (10) — 1 (10) = 2 (36) — 7 (10).

Следовательно, значение x, которое мы ищем, равно 2, а значение y, которое мы ищем, равно -7, и, конечно же, с этими значениями мы имеем 2 = 36x + 10y.

Этот метод отслеживания шагов, используемых для нахождения НОД двух целых чисел, всегда можно использовать для нахождения значений целых чисел x и y в теореме 1.

Теперь предположим, что нам дано сравнение:

предполагая, что a и p взаимно просты, мы можем найти два целых числа s и t такие, что as + pt = 1. Если мы возьмем это сравнение по модулю p, мы получим:

, что означает, что

Следовательно, если мы установим x в значение bs, мы найдем значение x, которое решает сравнение (*) выше.

Остается разработать алгоритм для решения сравнений, где модуль является простым числом, и уметь находить значения x и y в уравнении (1) выше.Для этого мы используем алгоритм Евклида. Вместо того, чтобы описывать, что происходит в целом, я проиллюстрирую несколько типичных примеров.

Предположим, кто-то хочет решить сравнение:

В качестве первого шага найдите наибольший общий делитель 23 и 11 с помощью алгоритма Евклида:

23 = 2 (11) +1

11 = 1 (11) + 0, поэтому НОД равен 1.

Из уравнения выше мы видим, что 1 = (1) (23) + (-2) 11. По модулю 23 это уравнение дает (-2) (11) ≡ 1 mod 23.Таким образом, x = -2 решает сравнение. Поскольку мы хотим иметь возможность записать ответ как значение от 0 до 22, мы видим, что -2 ≡ 21 mod 23, поэтому x = 21 является решением.

Теперь рассмотрим сравнение:

В качестве первого шага мы решим сравнение: 16x ≡ 1 mod 29

У нас есть 29 = 1 (16) +13 (2)

16 = 1 (13) + 3 (3)

13 = 4 ( 3) + 1. (4)

Итак, мы можем написать, что 1 = 1 (13) + (-4) (3).

Подставляя в это уравнение, используя тот факт, что из (3) 3 = 16-1 (13), получаем:

1 = 1 (13) + (-4) (16-1 (13)) = 5 ( 13) + (-4) (16) (5)

Теперь, используя (2) (из которого мы знаем, что 13 = 29-1 (16)), мы можем заменить 13 в (5):

1 = 5 (29-1 (16)) + (-4) (16) = (-9) (16) + (5) (29).

Последнее уравнение по модулю 29 дает:

Итак, значение x, которое мы ищем, равно -9, что сравнимо по модулю 29 с 20.

Поскольку теперь мы знаем, что 16 (20) сравнимо с 1 по модулю 29, мы можем умножить обе стороны этого сравнения на 5, чтобы получить 100 (16) сравнимо с 5 по модулю 29.Поскольку 100 сравнимо с 13 по модулю 29, мы заключаем, что 13 является решением сравнения, которое мы хотели решить:

Вы должны убедиться, что 16 (13) — 5 действительно оставляет нулевой остаток при делении на 29.

Приведенные выше методы можно расширить для решения сравнений, где p также не является простым числом. Преимущество работы с простым модулем состоит в том, что до тех пор, пока a в (*) не кратно p, мы всегда можем найти уникальное решение сравнения со значением x, имеющим значение от 0 до (p-1) .

Задачи:

1. Используйте алгоритм Евклида, чтобы найти НОД 27 и 42.

2. Используйте алгоритм Евклида, чтобы найти НОД 146 и 224.

3. Решите сравнение 21x ≡ 5 по модулю 29

4. Решите сравнение 31x ≡ 5 по модулю 23

5. Решите сравнение 16x ≡ 5 по модулю 23

Введение в теорию чисел

Раздел 5.2 Введение в теорию чисел

Мы использовали натуральные числа для решения задач.Это был правильный набор чисел для работы в дискретной математике, потому что мы всегда имели дело с целым рядом вещей. Натуральные числа были инструментом. Давайте теперь рассмотрим этот инструмент. Какие математические открытия мы можем сделать из самих натуральных чисел?

Это главный вопрос теории чисел: огромный, древний, сложный и, прежде всего, красивый раздел математики. Исторически теория чисел была известна как королева математики и в значительной степени была ветвью чистой математики , изученной ради нее самой, а не как средство понимания приложений реального мира.Однако это изменилось в последние годы, когда были обнаружены приложения теории чисел. Вероятно, наиболее известным примером этого является криптография RSA, один из методов, используемых для шифрования данных в Интернете. Это стало возможным благодаря теории чисел.

Какие вопросы относятся к области теории чисел? Вот показательный пример. Напомним, в нашем исследовании индукции мы спрашивали:

Какие суммы почтовых расходов можно произвести, используя только 5-центовые и 8-центовые марки?

Мы смогли доказать, что можно заработать на любую сумму, превышающую 27 центов.Вы можете задаться вопросом, что произойдет, если мы изменим номинал марок. Что, если бы у нас были марки 4 и 9 центов? Будет ли какая-то сумма, после которой все суммы будут возможны? Что ж, опять же, мы могли бы заменить две марки по 4 цента на марку по 9 центов или три марки по 9 центов на семь марок по 4 цента. В каждом случае мы можем создать еще один цент почтовой оплаты. Использование этого в качестве индуктивного случая позволит нам доказать, что может быть произведена любая сумма почтовых расходов, превышающая 23 цента.

Что, если бы у нас были марки по 2 и 4 цента.Здесь это выглядит менее многообещающим. Если мы возьмем некоторое количество марок по 2 цента и некоторое количество марок по 4 цента, что мы можем сказать об общей сумме? Может ли это быть странным? Не похоже.

Почему работают 5 и 8, 4 и 9 работают, а 2 и 4 не работают? Что такого в этих числах? Если бы я дал вам пару цифр, не могли бы вы сразу сказать, будут ли они работать или нет? Мы ответим на эти и другие вопросы, предварительно исследуя некоторые более простые свойства самих чисел.

Подраздел Делимость

Натуральные числа легко складывать и умножать. Если мы расширим наш фокус на все целые числа, то вычитание также будет простым (нам нужны отрицательные числа, чтобы мы могли вычесть любое число из любого другого числа, даже большего из меньшего). Разделение — первая операция, которая представляет собой сложную задачу. Если бы мы хотели расширить наш набор чисел, чтобы было возможно любое деление (возможно, исключая деление на 0), нам нужно было бы посмотреть на рациональные числа (набор всех чисел, которые можно записать как дроби).Это было бы перебором, поэтому мы откажемся от этого варианта.

На самом деле хорошо, что не каждое число можно разделить на другие числа. Это помогает нам понять структуру натуральных чисел и открывает двери для многих интересных вопросов и приложений.

Если заданы числа \ (a \) и \ (b \ text {,} \), возможно, что \ (a \ div b \) дает целое число. В этом случае мы говорим, что \ (b \) делит \ (a \ text {,} \) на символы, мы пишем \ (b \ mid a \ text {.} \) Если это верно, то \ (b \) является делителем или множителем \ (a \ text {,} \), а \ (a \) делится на \ (b \ text {.} \) In другими словами, если \ (b \ mid a \ text {,} \), то \ (a = bk \) для некоторого целого числа \ (k \) (это означает, что \ (a \) является некоторым кратным \ (b \ )).

Отношение делимости.

Даны целые числа \ (m \) и \ (n \ text {,} \), мы говорим «\ (m \) делит \ (n \)» и пишем

\ begin {уравнение *} м \ мид п \ end {уравнение *}

при условии, что \ (n \ div m \) является целым числом. Таким образом, следующие утверждения означают одно и то же:

  1. \ (\ Displaystyle м \ середина п \)
  2. \ (n = mk \) для некоторого целого числа \ (k \)
  3. \ (m \) является множителем (или делителем) \ (n \)
  4. \ (n \) делится на \ (m \ text {.} \)

Обратите внимание, что \ (m \ mid n \) — это инструкция. Это либо правда, либо ложь. С другой стороны, \ (n \ div m \) или \ (n / m \) — некоторое число. Если мы хотим заявить, что \ (n / m \) не является целым числом, поэтому \ (m \) не делит \ (n \ text {,} \), тогда мы можем написать \ (m \ nmid n \ text { .} \)

Пример 5.2.1.

Решите, истинно ли каждое из приведенных ниже утверждений.

  1. \ (\ Displaystyle 4 \ середина 20 \)
  2. \ (\ Displaystyle 20 \ середина 4 \)
  3. \ (\ Displaystyle 0 \ середина 5 \)
  4. \ (\ Displaystyle 5 \ середина 0 \)
  5. \ (\ Displaystyle 7 \ середина 7 \)
  6. \ (\ Displaystyle 1 \ середина 37 \)
  7. \ (\ Displaystyle -3 \ середина 12 \)
  8. \ (\ Displaystyle 8 \ середина 12 \)
  9. \ (\ Displaystyle 1642 \ середина 136299 \)
Решение
  1. Верно.4 «входит» в 20 пять раз без остатка. Другими словами, \ (20 \ div 4 = 5 \ text {,} \) целое число. Мы также могли бы оправдать это, сказав, что \ (20 \) кратно 4: \ (20 = 4 \ cdot 5 \ text {.} \)

  2. Ложь. Хотя 20 кратно 4, неверно, что \ (4 \) кратно 20.

  3. Ложь. \ (5 \ div 0 \) даже не определено, не говоря уже о целом числе.

  4. Верно. Фактически, \ (x \ mid 0 \) верно для всех \ (x \ text {.} \). Это потому, что 0 кратно каждому числу: \ (0 = x \ cdot 0 \ text {.} \)

  5. Верно. Фактически, \ (x \ mid x \) верно для всех \ (x \ text {.} \)

  6. Верно. 1 делит каждое число (кроме 0).

  7. Верно. Отрицательные числа отлично подходят для отношения делимости. Здесь \ (12 = -3 \ cdot 4 \ text {.} \) Также верно, что \ (3 \ mid -12 \) и \ (- 3 \ mid -12 \ text {.} \)

  8. Ложь. И 8, и 12 делятся на 4, но это не означает, что \ (12 \) делится на \ (8 \ text {.} \)

  9. Ложь.Увидеть ниже.

Этот последний пример поднимает вопрос: как можно решить, \ (m \ mid n \ text {?} \). Конечно, если бы у вас был надежный калькулятор, вы могли бы запросить у него значение \ (n \ div m \ text {.} \) Если он выдаст что-то, кроме целого числа, вы знаете \ (m \ nmid n \ text {.} \) Это похоже на читерство: у нас нет деления, поэтому мы должны действительно использовать деление для проверки делимости?

Хотя мы действительно не умеем делить, мы знаем, как умножать.Мы можем попытаться умножить \ (m \) на все большие и большие числа, пока не приблизимся к \ (n \ text {.} \) Насколько близко? Что ж, мы хотим быть уверены, что если мы умножим \ (m \) на следующее большее целое число, мы перейдем к \ (n \ text {.} \)

Например, давайте попробуем это решить, будет ли \ (1642 \ mid 136299 \ text {.} \) Начинать поиск кратных 1642:

\ begin {уравнение *} 1642 \ cdot 2 = 3284 \ qquad 1642 \ cdot 3 = 4926 \ qquad 1642 \ cdot 4 = 6568 \ qquad \ cdots \ text {.} \ end {уравнение *}

Все они намного меньше 136299.Полагаю, мы можем немного забежать вперед:

\ begin {уравнение *} 1642 \ cdot 50 = 82100 \ qquad 1642 \ cdot 80 = 131360 \ qquad 1642 \ cdot 85 = 139570 \ text {.} \ end {уравнение *}

А, значит, нам нужно искать где-то между 80 и 85. Попробуйте 83:

\ begin {уравнение *} 1642 \ cdot 83 = 136286 \ текст {.} \ end {уравнение *}

Это лучшее, что мы можем сделать? Насколько мы далеки от желаемого 136299? Если мы вычтем, мы получим \ (136299 — 136286 = 13 \ text {.} \). Итак, мы знаем, что не можем подняться до 84, это будет слишком много.Другими словами, мы обнаружили, что

\ begin {уравнение *} 136299 = 83 \ cdot 1642 + 13 \ text {.} \ end {уравнение *}

Так как \ (13 \ lt 1642 \ text {,} \) теперь мы можем с уверенностью сказать, что \ (1642 \ nmid 136299 \ text {.} \)

Оказывается, процесс, который мы прошли выше, можно повторить для любой пары чисел. Мы всегда можем записать число \ (a \) как некоторое кратное числу \ (b \) плюс некоторый остаток. Мы знаем это, потому что знаем о делении с остатком от начальной школы.Это просто способ выразить это умножением. Из-за процедурного характера, который может использоваться для нахождения остатка, этот факт обычно называют алгоритмом деления :

Алгоритм деления.

Для любых двух целых чисел \ (a \) и \ (b \ text {,} \) мы всегда можем найти целое число \ (q \) такое, что

\ begin {уравнение *} а = qb + r \ end {уравнение *}

, где \ (r \) — целое число, удовлетворяющее \ (0 \ le r \ lt | b | \)

Идея состоит в том, что мы всегда можем взять достаточно большое число, кратное \ (b \), чтобы остаток \ ( г \) как можно меньше.Мы допускаем возможность \ (r = 0 \ text {,} \), и в этом случае мы имеем \ (b \ mid a \ text {.} \)

Подраздел Остальные классы

Алгоритм деления сообщает нам, что при делении на \ (b \ text {.} \) Возможны только \ (b \) остатки. Если мы исправим этот делитель, мы сможем сгруппировать целые числа по остатку. Каждая группа называется классом остатка по модулю \ (b \) (или иногда классом остатка ).

Пример 5.2.2.

Опишите классы остатка по модулю \ (5 \ text {.} \)

Решение

Мы хотим классифицировать числа по тому, каким будет их остаток при делении на \ (5 \ text {.} \) Из алгоритма деления мы знаем, что будет ровно 5 классов остатка, потому что есть только 5 вариантов того, что \ ( r \) может быть (\ (0 \ le r \ lt 5 \)).

Сначала рассмотрим \ (r = 0 \ text {.} \) Здесь мы ищем все числа, делящиеся на \ (5 \), поскольку \ (a = 5q + 0 \ text {.} \) Другими словами, кратно 5. Получаем бесконечное множество

\ begin {уравнение *} \ {\ ldots, -15, -10, -5, 0, 5, 10, 15, 20, \ ldots \} \ text {.} \ end {уравнение *}

Обратите внимание, что мы также включаем отрицательные целые числа.

Затем рассмотрим \ (r = 1 \ text {.} \) Какие целые числа при делении на 5 дают остаток 1? Ну, конечно, 1, делает, как 6, и 11. Отрицательные? Здесь мы должны быть осторожны: \ (- 6 \) НЕ имеет остатка 1. Мы можем написать \ (- 6 = -2 \ cdot 5 + 4 \) или \ (- 6 = -1 \ cdot 5 — 1 \ text {,} \), но только один из них является «правильным» примером алгоритма деления: \ (r = 4 \), поскольку нам нужно, чтобы \ (r \) было неотрицательным. Фактически, чтобы получить \ (r = 1 \ text {,} \), мы должны иметь \ (- 4 \ text {,} \) или \ (- 9 \ text {,} \) и т. Д.Таким образом получаем остаток класса

\ begin {уравнение *} \ {\ ldots, -14, -9, -4, 1, 6, 11, 16, 21, \ ldots \} \ text {.} \ end {уравнение *}

Осталось еще три. Остальные классы для \ (2 \ text {,} \) \ (3 \ text {,} \) и \ (4 \) составляют, соответственно,

\ begin {уравнение *} \ {\ ldots, -13, -8, -3, 2, 7, 12, 17, 22, \ ldots \} \ end {уравнение *}

\ begin {уравнение *} \ {\ ldots, -12, -7, -2, 3, 8, 13, 18, 23, \ ldots \} \ end {уравнение *}

\ begin {уравнение *} \ {\ ldots, -11, -6, -1, 4, 9, 14, 19, 24, \ ldots \} \ text {.} \ end {уравнение *}

Обратите внимание, что в приведенном выше примере каждое целое число находится ровно в одном классе остатка. Технически это можно сказать, что классы остатка по модулю \ (b \) образуют разбиение целых чисел. 1 Самым важным фактом о разделах является то, что из раздела можно определить отношение эквивалентности : это отношение между парами чисел, которое действует во всех важных направлениях, например, в отношении «равных». 2

Можно разработать математическую теорию разбиений, доказать утверждения обо всех разбиениях в целом, а затем применить эти наблюдения к нашему случаю.

Опять же, существует математическая теория отношений эквивалентности, которая применима во многих других случаях, чем тот, который мы рассматриваем здесь.

Если отбросить забавный технический язык, идея действительно проста. Если два числа принадлежат одному и тому же классу остатка, то в некотором роде они совпадают.То есть они одинаковые до деления на \ (b \) . В случае, когда \ (b = 5 \) выше, числа \ (8 \) и \ (23 \ text {,} \), хотя и не одно и то же число, одинаковы, когда дело доходит до деления на 5, потому что оба есть остаток \ (3 \ text {.} \)

Имеет значение, что такое делитель: \ (8 \) и \ (23 \) одинаковы до деления на \ (5 \ text {,} \), но не до деления на \ (7 \ text {,} \), поскольку остаток \ (8 \) при делении на 7 равен 1, а остаток 23 — 2.

Имея это в виду, введем некоторые обозначения.Мы хотим сказать, что \ (8 \) и 23 в основном одинаковы, хотя и не равны. Было бы неправильно сказать \ (8 = 23 \ text {.} \) Вместо этого мы пишем \ (8 \ Equiv 23 \ text {.} \). Но это не всегда верно. Это работает, если мы думаем о делении на 5, поэтому нам нужно как-то обозначить это. На самом деле мы напишем следующее:

\ begin {уравнение *} 8 \ эквив 23 \ pmod {5} \ end {уравнение *}

, что читается как «8 сравнимо с 23 по модулю 5» (или просто «по модулю 5»). Конечно, тогда мы могли наблюдать, что

\ begin {уравнение *} 8 \ not \ Equiv 23 \ pmod {7} \ text {.} \ end {уравнение *}

Конгруэнтность по модулю \ (n \).

Мы говорим, что \ (a \) конгруэнтно \ (b \) по модулю \ (n \) , и пишем

\ begin {уравнение *} а \ эквив б \ пмод {п} \ end {уравнение *}

при условии, что \ (a \) и \ (b \) имеют одинаковый остаток при делении на \ (n \ text {.} \) Другими словами, при условии, что \ (a \) и \ (b \) принадлежат одному и тому же класс остатка по модулю \ (n \ text {.} \)

Во многих книгах сравнение по модулю \ (n \) определяется несколько иначе. Они говорят, что \ (a \ Equiv b \ pmod {n} \) тогда и только тогда, когда \ (n \ mid a-b \ text {.} \) Другими словами, два числа конгруэнтны по модулю \ (n \ text {,} \), если их разность кратна \ (n \ text {.} \). Итак, какое определение является правильным? Оказывается, это не имеет значения: они равнозначны.

Чтобы понять, почему, рассмотрим два числа \ (a \) и \ (b \), которые конгруэнтны по модулю \ (n \ text {.} \). Тогда \ (a \) и \ (b \) имеют одинаковый остаток, когда делится на \ (n \ text {.} \) У нас есть

\ begin {уравнение *} a = q_1 n + r \ qquad \ qquad b = q_2 n + r \ text {.} \ end {уравнение *}

Здесь два \ (r \) действительно совпадают.Рассмотрим, что мы получим, если возьмем разницу между \ (a \) и \ (b \ text {:} \)

\ begin {уравнение *} a-b = q_1n + r — (q_2n + r) = q_1n — q_2 n = (q_1-q_2) n \ text {.} \ end {уравнение *}

Итак, \ (a-b \) кратно \ (n \ text {,} \) или, что эквивалентно, \ (n \ mid a-b \ text {.} \)

С другой стороны, если мы сначала предположим, что \ (n \ mid ab \ text {,} \) so \ (ab = kn \ text {,} \), то рассмотрим, что произойдет, если мы разделим каждый член на \ (n \ text {.} \) При делении \ (a \) на \ (n \) останется некоторый остаток, как и при делении \ (b \) на \ (n \ text {.} \) Однако деление \ (kn \) на \ (n \) оставит 0 остатка. Таким образом, остатки в левой части должны уравняться. То есть остатки должны быть такими же.

Таким образом имеем:

Конгруэнтность и делимость.

Для любых целых чисел \ (a \ text {,} \) \ (b \ text {,} \) и \ (n \ text {,} \) имеем

\ begin {уравнение *} a \ Equiv b \ pmod {n} \ qquad \ text {тогда и только тогда, когда} \ qquad n \ mid a-b \ text {.} \ end {уравнение *}

Также будет полезно переключаться между сравнениями и регулярными уравнениями.Приведенный выше факт помогает в этом. Мы знаем, что \ (a \ Equiv b \ pmod {n} \) тогда и только тогда, когда \ (n \ mid ab \ text {,} \) тогда и только тогда, когда \ (ab = kn \) для некоторого целого числа \ (k \ text {.} \) Переставляя это уравнение, мы получаем \ (a = b + kn \ text {.} \) Другими словами, если \ (a \) и \ (b \) конгруэнтны по модулю \ (n \ text {,} \), то \ (a \) на \ (b \) больше, чем некоторое кратное \ (n \ text {.} \). Это согласуется с нашим предыдущим наблюдением, что все числа в конкретном классе остатка являются такое же количество больше, чем кратное \ (n \ text {.} \)

Конгруэнтность и равенство.

Для любых целых чисел \ (a \ text {,} \) \ (b \ text {,} \) и \ (n \ text {,} \) имеем

\ begin {уравнение *} a \ Equiv b \ pmod {n} \ qquad \ text {тогда и только тогда, когда} \ qquad a = b + kn \ mbox {для некоторого целого числа} k \ text {.} \ end {уравнение *}

Подраздел Свойства сравнения

Ранее мы говорили, что сравнение по модулю \ (n \) во многих важных отношениях ведет себя так же, как и равенство. В частности, мы могли бы доказать, что сравнение по модулю \ (n \) является отношением эквивалентности , что потребует проверки следующих трех фактов:

Конгруэнтность по модулю \ (n \) является отношением эквивалентности.

Для любых целых чисел \ (a \ text {,} \) \ (b \ text {,} \) и \ (c \ text {,} \) и любого положительного целого числа \ (n \ text {,} \) следующий трюм:

  1. \ (а \ эквивалент а \ pmod {n} \ text {.} \)
  2. Если \ (a \ Equiv b \ pmod {n} \), то \ (b \ Equiv a \ pmod {n} \ text {.} \)

  3. Если \ (a \ Equiv b \ pmod {n} \) и \ (b \ Equiv c \ pmod {n} \ text {,} \), то \ (a \ Equiv c \ pmod {n} \ text { .} \)

Другими словами, сравнение по модулю \ (n \) рефлексивно, симметрично и транзитивно, как и отношение эквивалентности.

Уделите минуту, чтобы убедиться, что каждое из вышеперечисленных свойств действительно соответствует друг другу. Попробуйте объяснить каждый из них, используя определения как остатка, так и делимости.

Затем рассмотрим, как ведет себя конгруэнтность при выполнении основных арифметических действий. Мы уже знаем, что если вы вычесть два конгруэнтных числа, результат будет сравним с 0 (кратным \ (n \)). Что, если мы добавим что-то, совпадающее с 1, к чему-то, совпадающему с 2? Получим ли мы что-нибудь похожее на 3?

Сравнение и арифметика.

Предположим, что \ (a \ Equiv b \ pmod {n} \) и \ (c \ Equiv d \ pmod {n} \ text {.} \), Тогда выполняется следующее:

  1. \ (a + c \ Equiv b + d \ pmod {n} \ text {.} \)
  2. \ (a-c \ Equiv b-d \ pmod {n} \ text {.} \)
  3. \ (ac \ Equiv bd \ pmod {n} \ text {.} \)

Приведенные выше факты могут быть написаны немного странно, но идея проста. Если у нас есть истинное совпадение, и мы добавляем одно и то же к обеим сторонам, результатом все равно будет истинное совпадение. Похоже, мы говорим:

Если \ (a \ Equiv b \ pmod {n} \), то \ (a + c \ Equiv b + c \ pmod {n} \ text {.} \)

Конечно, это также верно, это частный случай, когда \ (c = d \ text {.} \). Но то, что у нас есть, работает в более общем смысле. Думайте о конгруэнтности как о «практически равном». Если у нас есть два числа, которые в основном равны, и мы добавляем в основном одно и то же к обеим сторонам, результат будет в основном одинаковым.

Это кажется разумным. Это правда правда? Докажем первый факт:

Доказательство.

Предположим, что \ (a \ Equiv b \ pmod {n} \) и \ (c \ Equiv d \ pmod {n} \ text {.} \) Это означает \ (a = b + kn \) и \ (c = d + jn \) для целых чисел \ (k \) и \ (j \ text {.} \) Добавьте эти уравнения:

\ begin {уравнение *} a + c = b + d + kn + jn \ text {.} \ end {уравнение *}

Но \ (kn + jn = (k + j) n \ text {,} \), которое просто кратно \ (n \ text {.} \) Итак \ (a + c = b + d + (j + k) n \ text {,} \) или, другими словами, \ (a + c \ Equiv b + d \ pmod {n} \)

Два других факта можно доказать аналогичным образом.

Одним из важных следствий этих фактов о сравнениях является то, что мы можем в принципе заменить любое число в сравнении любым другим числом, с которым оно конгруэнтно.Вот несколько примеров, чтобы увидеть, как (и почему) это работает:

Пример 5.2.3.

Найдите остаток от деления \ (3491 \) на \ (9 \ text {.} \)

Решение

Мы могли бы делать деление в длину, но есть другой способ. Мы хотим найти \ (x \) такой, что \ (x \ Equiv 3491 \ pmod {9} \ text {.} \) Теперь \ (3491 = 3000 + 400 + 90 + 1 \ text {.} \) Конечно \ (90 \ Equiv 0 \ pmod 9 \ text {,} \), поэтому мы можем заменить 90 в сумме на 0. Почему это нормально? Фактически мы вычитаем «одно и то же» с обеих сторон:

\ begin {уравнение *} \ begin {align} x \ amp \ эквив 3000 + 400 + 90 + 1 \ pmod 9 \\ — ~~ 0 \ amp \ эквив 90 \ pmod 9 \\ х \ amp \ эквив 3000 + 400 + 0 + 1 \ pmod 9.\ end {выровнен} \ end {уравнение *}

Затем обратите внимание, что \ (400 = 4 \ cdot 100 \ text {,} \) и \ (100 \ Equiv 1 \ pmod 9 \) (поскольку \ (9 \ mid 99 \)). Таким образом, мы можем фактически заменить 400 на просто 4. Мы снова апеллируем к нашему утверждению, что мы можем заменять конгруэнтные элементы, но на самом деле мы обращаемся к свойству 3 об арифметике сравнения: мы знаем \ (100 \ эквив 1 \ pmod {9} \ text {,} \), поэтому, если мы умножим обе стороны на \ (4 \ text {,} \), мы получим \ (400 \ Equiv 4 \ pmod 9 \ text {.} \)

Точно так же мы можем заменить 3000 на 3, так как \ (1000 = 1 + 999 \ эквив 1 \ pmod 9 \ text {.} \) Таким образом, наше исходное сравнение становится

\ begin {уравнение *} х \ эквив 3 + 4 + 0 + 1 \ pmod 9 \ end {уравнение *}

\ begin {уравнение *} х \ экви 8 \ pmod 9 \ текст {.} \ end {уравнение *}

Следовательно, \ (3491 \), деленное на 9, имеет остаток 8.

Приведенный выше пример должен убедить вас, что хорошо известный тест делимости числа 9 верен: сумма цифр числа делится на 9 тогда и только тогда, когда исходное число делится на 9. Фактически, теперь мы знаем кое-что еще. : любое число конгруэнтно сумме своих цифр по модулю 9.p \ pmod n \ text {.} \) Это просто применение свойства 3 несколько раз.

До сих пор мы видели, как складывать, вычитать и умножать с помощью сравнений. А как насчет разделения? Есть причина, по которой мы ждали, чтобы обсудить это. Получается, что просто разделить нельзя. Другими словами, даже если \ (ad \ Equiv bd \ pmod n \ text {,} \) мы не знаем, что \ (a \ Equiv b \ pmod n \ text {.} \) Рассмотрим, например:

\ begin {уравнение *} 18 \ экви 42 \ pmod 8 \ текст {.} \ end {уравнение *}

Это правда.Теперь \ (18 \) и \ (42 \) делятся на 6. Однако

\ begin {уравнение *} 3 \ not \ Equiv 7 \ pmod 8 \ text {.} \ end {уравнение *}

Хотя это не работает, обратите внимание, что \ (3 \ Equiv 7 \ pmod 4 \ text {.} \) Мы не можем разделить \ (8 \) на 6, но мы можем разделить 8 на наибольший общий делитель \ ( 8 \) и \ (6 \ text {.} \) Всегда ли это будет?

Предположим, \ (ad \ Equiv bd \ pmod n \ text {.} \) Другими словами, у нас есть \ (ad = bd + kn \) для некоторого целого числа \ (k \ text {.} \) Конечно \ ( ad \) делится на \ (d \ text {,} \), как и \ (bd \ text {.} \) Итак, \ (kn \) также должно делиться на \ (d \ text {.} \). Если \ (n \) и \ (d \) не имеют общих делителей (кроме 1), то мы должны иметь \ (d \ mid k \ text {.} \) Но в целом, если мы попытаемся разделить \ (kn \) на \ (d \ text {,} \), мы не знаем, что получим целое число кратное \ (n \ text {.} \) Некоторые из \ (n \) также могут разделиться. На всякий случай разделим как можно больше \ (n \). Возьмите наибольший множитель как \ (d \), так и \ (n \ text {,} \) и исключите его из \ (n \ text {.} \) Остальные множители \ (d \) придут из \ (k \ text {,} \) без проблем.

Мы будем называть наибольший множитель как \ (d \), так и \ (n \) \ (\ gcd (d, n) \ text {,} \) для наибольшего общего делителя . В нашем примере выше \ (\ gcd (6,8) = 2 \), поскольку наибольший делитель, общий для 6 и 8, равен 2.

Конгруэнтность и деление.

Предположим, \ (ad \ Equiv bd \ pmod n \ text {.} \) Затем \ (a \ Equiv b \ pmod {\ frac {n} {\ gcd (d, n)}} \ text {.} \)

Если \ (d \) и \ (n \) не имеют общих множителей, то \ (\ gcd (d, n) = 1 \ text {,} \), поэтому \ (a \ Equiv b \ pmod n \ text {. } \)

Пример 5.2.5.

Упростите следующие сравнения с помощью деления: (a) \ (24 \ Equiv 39 \ pmod 5 \) и (b) \ (24 \ Equiv 39 \ pmod {15} \ text {.} \)

Решение

(a) И \ (24 \), и \ (39 \) делятся на \ (3 \ text {,} \), а \ (3 \) и \ (5 \) не имеют общих делителей, поэтому мы получаем

\ begin {уравнение *} 8 \ Equiv 13 \ pmod 5 \ text {.} \ end {уравнение *}

(b) Опять же, мы можем разделить на 3. Однако, делая это вслепую, мы получаем \ (8 \ Equiv 13 \ pmod {15} \), что больше не соответствует действительности. Вместо этого мы также должны разделить модуль 15 на наибольший общий делитель \ (3 \) и \ (15 \ text {,} \), который равен \ (3 \ text {.} \) Снова получаем

\ begin {уравнение *} 8 \ Equiv 13 \ pmod 5 \ text {.} \ end {уравнение *}

Подраздел Решение сравнений

Теперь, когда у нас есть некоторые алгебраические правила для управления отношениями сравнения, мы можем попытаться найти неизвестное в сравнении. Например, существует ли значение \ (x \), которое удовлетворяет,

\ begin {уравнение *} 3x + 2 \ Equiv 4 \ pmod {5} \ text {,} \ end {уравнение *}

и если да, то что это?

В этом примере, поскольку модуль мал, мы могли бы просто попробовать все возможные значения для \ (x \ text {.} \) На самом деле нужно рассмотреть только 5, поскольку любое целое число, удовлетворяющее конгруэнтности, может быть заменено любым другим целым числом, совпадающим с модулем 5. Здесь, когда \ (x = 4 \), мы получаем \ (3x + 2 = 14 \), что действительно сравнимо с 4 по модулю 5. Это означает, что \ (x = 9 \), \ (x = 14 \) и \ (x = 19 \) и т.д. каждое также будет решением, потому что, поскольку мы Как было показано выше, замена любого числа в сравнении на совпадающее не меняет истинности сравнения.

Итак, в этом примере просто вычислите \ (3x + 2 \) для значений \ (x \ in \ {0,1,2,3,4 \} \ text {.} \) Это дает 2, 5, 8, 11 и 14 соответственно, из которых только 14 конгруэнтно 4.

Давайте также посмотрим, как вы можете решить эту проблему, используя наши правила алгебры сравнений. Такой подход был бы намного проще, чем метод проб и ошибок, если бы модуль упругости был больше. Во-первых, мы знаем, что можем вычесть 2 из обеих частей:

\ begin {уравнение *} 3x \ эквив 2 \ pmod {5} \ текст {.} \ end {уравнение *}

Затем, чтобы разделить обе стороны на 3, мы сначала прибавляем 0 к обеим сторонам. Конечно, в правой части мы хотим, чтобы 0 был равен 10 (да, \ (10 ​​\) действительно равно 0, поскольку они конгруэнтны по модулю 5).Это дает,

\ begin {уравнение *} 3x \ эквив 12 \ pmod {5} \ текст {.} \ end {уравнение *}

Теперь разделите обе стороны на 3. Поскольку \ (\ gcd (3,5) = 1 \ text {,} \) нам не нужно изменять модуль:

\ begin {уравнение *} х \ эквив 4 \ pmod {5} \ текст {.} \ end {уравнение *}

Обратите внимание, что это фактически дает общее решение : не только может \ (x = 4 \ text {,} \), но и \ (x \) может быть любым числом, которое конгруэнтно 4. Мы можем оставить это так. , или напишите «\ (x = 4 + 5k \) для любого целого числа \ (k \ text {.} \) ”

Пример 5.2.6.

Решите следующие сравнения для \ (x \ text {.} \)

  1. \ (7x \ Equiv 12 \ pmod {13} \ text {.} \)
  2. \ (84x — 38 \ эквив 79 \ pmod {15} \ text {.} \)
  3. \ (20x \ Equiv 23 \ pmod {14} \ text {.} \)
Решение
  1. Все, что нам нужно здесь сделать, это разделить обе части на 7. Мы прибавляем 13 к правой части несколько раз, пока не получим число, кратное 7 (добавление 13 аналогично добавлению 0, так что это допустимо). Получаем \ (25 \ text {,} \) \ (38 \ text {,} \) \ (51 \ text {,} \) \ (64 \ text {,} \) \ (77 \) — получили .Итак, имеем:

    \ begin {уравнение *} \ begin {align} 7x \ amp \ эквив 12 \ pmod {13} \\ 7x \ amp \ эквив 77 \ pmod {13} \\ х \ amp \ эквив 11 \ pmod {13}. \ end {выровнен} \ end {уравнение *}

  2. Здесь, поскольку у нас есть числа, превышающие модуль, мы можем уменьшить их до применения какой-либо алгебры. У нас есть \ (84 \ эквив 9 \ текст {,} \) \ (38 \ эквив 8 \) и \ (79 \ эквив 4 \ текст {.} \) Таким образом,

    \ begin {уравнение *} \ begin {align} 84x — 38 \ amp \ эквив 79 \ pmod {15} \\ 9x — 8 \ amp \ эквив 4 \ pmod {15} \\ 9x \ amp \ эквив 12 \ pmod {15} \\ 9x \ amp \ эквив 72 \ pmod {15}.\ end {выровнен} \ end {уравнение *}

    Мы получили 72, добавив \ (0 \ Equiv 60 \ pmod {15} \) к обеим сторонам сравнения. Теперь разделите обе стороны на 9. Однако, поскольку \ (\ gcd (9, 15) = 3 \ text {,} \), мы также должны разделить модуль на 3:

    \ begin {уравнение *} х \ экви 8 \ pmod 5 \ текст {.} \ end {уравнение *}

    Таким образом, решения — это те значения, которые сравнимы с 8 или, что эквивалентно 3, по модулю 5. Это означает, что в некотором смысле есть 3 решения по модулю 15: 3, 8 и 13. Мы можем записать решение:

    \ begin {уравнение *} х \ эквив 3 \ pmod {15}; ~~ х \ экв 8 \ pmod {15}; ~~ х \ эквив 13 \ pmod {15} \ text {.} \ end {уравнение *}

  3. Сначала уменьшаем по модулю 14:

    \ begin {уравнение *} 20x \ эквив 23 \ pmod {14} \ end {уравнение *}

    \ begin {уравнение *} 6x \ Equiv 9 \ pmod {14} \ text {.} \ end {уравнение *}

    Теперь мы можем разделить обе стороны на 3 или попытаться увеличить 9 на 14, чтобы получить число, кратное 6. Если мы разделим на 3, мы получим

    \ begin {уравнение *} 2x \ Equiv 3 \ pmod {14} \ text {.} \ end {уравнение *}

    Теперь попробуйте сложить число, кратное 14, к 3, в надежде получить число, которое мы можем разделить на 2.Так не пойдет! Каждый раз, когда мы добавляем 14 к правой части, результат все равно будет нечетным. У нас никогда не будет четного числа, поэтому мы никогда не сможем разделить на 2. Таким образом, у сравнения нет решений.

Последнее сравнение выше показывает, как сравнения могут не иметь решений. Фактически, мы могли это сразу увидеть. Посмотрите на исходное соответствие:

\ begin {уравнение *} 20x \ эквив 23 \ pmod {14} \ text {.} \ end {уравнение *}

Если мы запишем это в виде уравнения, мы получим

\ begin {уравнение *} 20x = 23 + 14k \ text {,} \ end {уравнение *}

или эквивалентно \ (20x — 14k = 23 \ text {.} \) Легко видеть, что у этого уравнения не будет решения в целых числах. Левая часть всегда будет четной, а правая — нечетной. Аналогичная проблема возникла бы, если бы правая часть делилась на любого числа , которого не было в левой части.

Так в целом с учетом сравнения

\ begin {уравнение *} ах \ эквив б \ пмод {п} \ текст {,} \ end {уравнение *}

, если \ (a \) и \ (n \) делятся на число, на которое не делится \ (b \), то решений не будет.Фактически, нам действительно нужно проверить только один делитель \ (a \) и \ (n \ text {:} \) наибольший общий делитель. Таким образом, более компактно это сказать:

Конгруэнции без решений.

Если \ (\ gcd (a, n) \ nmid b \ text {,} \), то \ (ax \ Equiv b \ pmod {n} \) не имеет решений.

Подраздел Решение линейных диофантовых уравнений

Дискретная математика имеет дело с целым рядом вещей. Поэтому, когда мы хотим решить уравнения, мы обычно ищем целочисленных решений .2 \ text {.} \) Целочисленные решения этого уравнения называются пифагоровыми тройками . В общем, решение диофантовых уравнений сложно (на самом деле, доказуемо не существует общего алгоритма для определения того, имеет ли диофантово уравнение решение, результат, известный как теорема Матиясевича). Мы ограничимся рассмотрением линейных и диофантовых уравнений, с которыми значительно легче работать.

Диофантовы уравнения.

Уравнение с двумя или более переменными называется диофантовым уравнением , если интерес представляют только целочисленные решения.Линейное диофантово уравнение принимает вид \ (a_1x_1 + a_2x_x + \ cdots + a_nx_n = b \) для констант \ (a_1, \ ldots, a_n, b \ text {.} \)

Решение диофантова уравнения — это решение уравнения, состоящее только из целых чисел.

У нас есть инструменты, необходимые для решения линейных диофантовых уравнений. Рассмотрим в качестве основного примера уравнение

\ begin {уравнение *} 51x + 87y = 123 \ текст {.} \ end {уравнение *}

Общая стратегия состоит в том, чтобы преобразовать уравнение в сравнение, а затем решить это сравнение. 4 Давайте поработаем на этом конкретном примере, чтобы увидеть, как это может происходить.

Это, конечно, не единственный способ продолжить. Более распространенным методом было бы применение алгоритма Евклида . Наш путь может быть немного быстрее, и он представлен здесь в первую очередь для разнообразия.

Сначала проверьте, возможно, нет решений, потому что делитель \ (51 \) и \ (87 \) не является делителем \ (123 \ text {.} \). На самом деле, нам просто нужно проверить, \ ( \ gcd (51, 87) \ середина 123 \ текст {.} \) Этот наибольший общий делитель равен 3, и да \ (3 \ mid 123 \ text {.} \) На этом этапе мы могли бы также вычесть этот наибольший общий делитель. Вместо этого мы решим:

\ begin {уравнение *} 17x + 29y = 41 \ текст {.} \ end {уравнение *}

Теперь заметьте, что если будут решения, то для этих значений \ (x \) и \ (y \ text {,} \) две стороны уравнения должны иметь одинаковый остаток друг от друга, независимо от на что мы делим. В частности, если мы разделим обе части на 17, мы должны получить одинаковый остаток.Таким образом, мы можем смело писать

\ begin {уравнение *} 17x + 29y \ эквив 41 \ pmod {17} \ text {.} \ end {уравнение *}

Мы выбрали 17, потому что \ (17x \) будет иметь остаток 0. Это позволит нам уменьшить сравнение до одной переменной. Мы также могли бы перейти к сравнению по модулю 29, хотя обычно есть веская причина выбрать меньший вариант, так как это позволит нам уменьшить другой коэффициент. В нашем случае мы сокращаем сравнение следующим образом:

\ begin {уравнение *} \ begin {align} 17x + 29y \ amp \ Equiv 41 \ pmod {17} \\ 0x + 12y \ amp \ эквив 7 \ pmod {17} \\ 12 лет \ amp \ эквив 24 \ pmod {17} \\ у \ amp \ эквив 2 \ pmod {17}.\ end {выровнен} \ end {уравнение *}

Теперь мы знаем, что \ (y = 2 + 17k \) будет работать для любого целого числа \ (k \ text {.} \). Если мы не ошиблись, мы сможем снова подключить это к нашему исходное диофантово уравнение, чтобы найти \ (x \ text {:} \)

\ begin {уравнение *} \ begin {align} 17x + 29 (2 + 17k) \ amp = 41 \\ 17x \ amp = -17 — 29 \ cdot 17k \\ х \ amp = -1-29к. \ end {выровнен} \ end {уравнение *}

Мы нашли все решения диофантова уравнения. Для каждого \ (k \ text {,} \) \ (x = -1-29k \) и \ (y = 2 + 17k \) будут удовлетворять уравнению.Мы можем проверить это в нескольких случаях. Если \ (k = 0 \ text {,} \), решением будет \ ((- 1,2) \ text {,} \) и да, \ (- 17 + 2 \ cdot 29 = 41 \ text {.} \) Если \ (k = 3 \ text {,} \), решение будет \ ((- 88, 53) \ text {.} \) Если \ (k = -2 \ text {,} \), мы получим \ ((57, -32) \ text {.} \)

Подводя итог этому процессу, для решения \ (ax + by = c \ text {,} \) мы,

  1. Разделите обе части уравнения на \ (\ gcd (a, b) \) (если это не оставляет правую часть в виде целого числа, решений нет). Предположим, что \ (ax + by = c \) уже уменьшено таким образом.

  2. Выберите меньшее из \ (a \) и \ (b \) (здесь, предположим, это \ (b \)), и преобразуйте его в сравнение по модулю \ (b \ text {:} \)

    \ begin {уравнение *} ax + автор \ Equiv c \ pmod {b} \ text {.} \ end {уравнение *}

    Это сведется к конгруэнтности с одной переменной, \ (x \ text {:} \)

    \ begin {уравнение *} ах \ эквив с \ pmod {Ь} \ текст {.} \ end {уравнение *}

  3. Решите сравнение, как мы делали в предыдущем разделе. Запишите свое решение в виде уравнения, например,

    \ begin {уравнение *} х = п + кб \ текст {.} \ end {уравнение *}

  4. Вставьте это в исходное диофантово уравнение и решите относительно \ (y \ text {.} \)

  5. Если мы хотим узнать решения в определенном диапазоне (например, \ (0 \ le x, y \ le 20 \)), выбирайте разные значения \ (k \), пока не получите все требуемые решения.

Вот еще пример:

Пример 5.2.7.

Как вы можете заработать 6,37 доллара, используя только 5-центовые и 8-центовые марки? Какое наименьшее и наибольшее количество марок вы могли бы использовать?

Решение

Во-первых, нам нужно диофантово уравнение.Мы будем работать в центах. Пусть \ (x \) — количество марок с ценой \ (5 \) центов, а \ (y \) — количество марок по 8 центов. У нас:

\ begin {уравнение *} 5x + 8y = 637 \ текст {.} \ end {уравнение *}

Преобразуйте в сравнение и решите:

\ begin {уравнение *} \ begin {align} 8y \ amp \ эквив 637 \ pmod {5} \\ 3у \ amp \ эквив 2 \ pmod 5 \\ 3у \ amp \ эквив 12 \ pmod 5 \\ у \ amp \ эквив 4 \ pmod 5. \ end {выровнен} \ end {уравнение *}

Таким образом, \ (y = 4 + 5k \ text {.} \) Тогда \ (5x + 8 (4 + 5k) = 637 \ text {,} \), поэтому \ (x = 121 — 8k \ text {.} \)

Здесь говорится, что один из способов заработать 6,37 доллара — это взять 121 из 5-центовых марок и 4 из 8-центовых марок. Чтобы найти наименьшее и наибольшее количество штампов, попробуйте разные значения \ (k \ text {.} \)

\ (к \) \ ((х, у) \) марок
–1 (129, -1) невозможно
0 (121, 4) 125
1 (113, 9) 122
2 (105, 13) 119
\ (\ vdots \) ​​ \ (\ vdots \) ​​ \ (\ vdots \) ​​

Это неудивительно.Наличие наибольшего количества марок означает, что у нас есть как можно больше 5-центовых марок, а для получения наименьшего количества марок потребуется наименьшее количество 5-центовых марок. Чтобы минимизировать количество 5-центовых марок, мы хотим выбрать \ (k \) так, чтобы \ (121-8k \) было как можно меньше (но все же положительным). Когда \ (k = 15 \ text {,} \) мы имеем \ (x = 1 \) и \ (y = 79 \ text {.} \)

Таким образом, чтобы заработать 6,37 доллара, вы можете использовать всего 80 марок (1 5-центовая марка и 79 8-центовых марок) или целых 125 марок (121 5-центовая и 4 8-центовых).

Используя этот метод, если вы можете решать линейные сравнения с одной переменной, вы можете решать линейные диофантовы уравнения с двумя переменными. Однако бывают случаи, когда решение линейного сравнения — это большая работа. Например, предположим, что вам нужно решить,

\ begin {уравнение *} 13x \ Equiv 6 \ pmod {51} \ text {.} \ end {уравнение *}

Вы, , могли бы продолжать прибавлять 51 к правой части, пока не получите число, кратное 13: вы получите 57, 108, 159, 210, 261, 312, и 312 — первое из них, которое делится на 13.Это работает, но на самом деле это слишком много. Вместо этого мы могли бы преобразовать обратно в диофантово уравнение:

\ begin {уравнение *} 13x = 6 + 51k \ text {.} \ end {уравнение *}

Теперь решите и этот , как в этом разделе. Запишите это как сравнение по модулю 13:

.

\ begin {уравнение *} \ begin {align} 0 \ amp \ эквив 6 + 51k \ pmod {13} \\ -12к \ amp \ эквив 6 \ pmod {13} \\ 2к \ amp \ эквив -1 \ pmod {13} \\ 2k \ amp \ эквив 12 \ pmod {13} \\ к \ amp \ эквив 6 \ pmod {13}. \ end {выровнен} \ end {уравнение *}

так что \ (k = 6 + 13j \ text {.} \) Теперь вернитесь и найдите \ (x \ text {:} \)

\ begin {уравнение *} \ begin {align} 13x \ amp = 6 + 51 (6 + 13j) \\ х \ amp = 24 + 51j. \ end {выровнен} \ end {уравнение *}

Конечно, вы можете переключаться между сравнениями и диофантовыми уравнениями столько раз, сколько захотите. Если бы вы только использовали эту технику, вы бы по существу воспроизвели алгоритм Евклида, более стандартный способ решения диофантовых уравнений.

Упражнения Упражнения

1.

Предположим, что \ (a \ text {,} \) \ (b \ text {,} \) и \ (c \) — целые числа. Докажите, что если \ (a \ mid b \ text {,} \), то \ (a \ mid bc \ text {.} \)

Решение
Доказательство.

Предположим, \ (a \ mid b \ text {.} \) Тогда \ (b \) кратно \ (a \ text {,} \) или, другими словами, \ (b = ak \) для некоторого \ (k \ text {.} \) Но тогда \ (bc = akc \ text {,} \) и поскольку \ (kc \) является целым числом, это означает, что \ (bc \) кратно \ (a \ text {.} \) Другими словами, \ (a \ mid bc \ text {.} \)

2.

Предположим, что \ (a \ text {,} \) \ (b \ text {,} \) и \ (c \) — целые числа.Докажите, что если \ (a \ mid b \) и \ (a \ mid c \), то \ (a \ mid b + c \) и \ (a \ mid b-c \ text {.} \)

3.

Запишите оставшиеся классы для \ (n = 4 \ text {.} \)

Решение

\ (\ {\ ldots, -8, -4, 0, 4, 8, 12, \ ldots \} \ text {,} \) \ (\ {\ ldots, -7, -3, 1, 5, 9, 13, \ ldots \} \ text {,} \)

\ (\ {\ ldots, -6, -2, 2, 6, 10, 14, \ ldots \} \ text {,} \) и \ (\ {\ ldots, -5, -1, 3, 7 , 11, 15, \ ldots \} \ text {.} \)

4.

Какой самый большой \ (n \) такой, что \ (16 \) и \ (25 \) находятся в одном классе остатка по модулю \ (n \ text {?} \). Запишите остаточный класс, к которому они оба принадлежат, и приведите пример числа более 100 в этом классе.

5.

Пусть \ (a \ text {,} \) \ (b \ text {,} \) \ (c \ text {,} \) и \ (n \) будут целыми числами. Докажите, что если \ (a \ Equiv b \ pmod {n} \) и \ (c \ Equiv d \ pmod {n} \ text {,} \), то \ (ac \ Equiv bd \ pmod {n} \ text { .} \)

Решение
Доказательство.

Предположим, \ (a \ Equiv b \ pmod n \) и \ (c \ Equiv d \ pmod n \ text {.} \). Это означает \ (a = b + kn \) и \ (c = d + jn \ ) для некоторых целых чисел \ (k \) и \ (j \ text {.} \) Рассмотрим \ (ac \ text {.} \) Имеем:

\ begin {уравнение *} а-с = Ь + кн — (д + jn) = Ь-д + (к-j) п \ текст {.2 \ Equiv 2 \ pmod 4 \ text {.} \) Решение

Для всего этого просто вставьте все целые числа от 0 до модуля, чтобы увидеть, какие из них работают.

  1. Нет решений.

  2. \ (x = 2 \ text {,} \) \ (x = 5 \ text {,} \) \ (x = 8 \ text {.} \)
  3. Нет решений.

9.

Определите, какие из следующих сравнений имеют решения, и найдите любые решения (от 0 до модуля) методом проб и ошибок.

  1. \ (4x \ эквив 5 \ pmod 7 \ text {.2 \ Equiv 2 \ pmod 7 \ text {.} \)
10.

Решите следующее сравнение \ (5x + 8 \ Equiv 11 \ pmod {22} \ text {.} \), То есть опишите общее решение.

Решение

\ (x = 5 + 22k \) для \ (k \ in \ Z \ text {.} \)

11.

Решите сравнение: \ (6x \ Equiv 4 \ pmod {10} \ text {.} \)

12.

Решите сравнение: \ (4x \ Equiv 24 \ pmod {30} \ text {.} \)

Решение

\ (x = 6 + 15k \) для \ (k \ in \ Z \ text {.} \)

13.

Решите сравнение: \ (341x \ Equiv 2941 \ pmod {9} \ text {.} \)

Подсказка

Сначала уменьшите каждое число по модулю 9, что можно сделать, сложив цифры чисел.

14.

Я думаю о числе. Если вы умножите мое число на 7, прибавите 5 и разделите результат на 11, у вас останется остаток 2. Какой остаток вы получите, если разделите мое исходное число на 11?

Решение

Мы должны решить \ (7x + 5 \ Equiv 2 \ pmod {11} \ text {.} \). Это дает \ (x \ Equiv 9 \ pmod {11} \ text {.} \) В общем, \ ( x = 9 + 11k \ text {,} \), но когда вы разделите любой такой \ (x \) на 11, остаток будет равен 9.

15.

Решите следующее линейное диофантово уравнение, используя модульную арифметику (опишите общие решения).

\ begin {уравнение *} 6x + 10y = 32 \ текст {.} \ end {уравнение *}

Решение

Разделить на 2: \ (3x + 5y = 16 \ text {.} \) Преобразовать в сравнение по модулю 3: \ (5y \ Equiv 16 \ pmod 3 \ text {,} \), которое сводится к \ ( 2y \ Equiv 1 \ pmod 3 \ text {.} \) Итак, \ (y \ Equiv 2 \ pmod 3 \) или \ (y = 2 + 3k \ text {.} \) Вставьте это обратно в \ (3x + 5y = 16 \) и решите для \ (x \ text {,} \), чтобы получить \ (x = 2-5k \ text {.} \) Таким образом, общее решение — \ (x = 2-5k \) и \ (y = 2 + 3k \) для \ (k \ in \ Z \ text {.} \)

16.

Решите следующее линейное диофантово уравнение, используя модульную арифметику (опишите общие решения).

\ begin {уравнение *} 17x + 8y = 31 \ текст {.} \ end {уравнение *}

17.

Решите следующее линейное диофантово уравнение, используя модульную арифметику (опишите общие решения).

\ begin {уравнение *} 35x + 47y = 1 \ текст {.} \ end {уравнение *}

18.

У вас есть 13 унций. бутылка и 20 унций. бутылка, с которой вы хотите отмерить ровно 2 унции. Однако у вас ограниченный запас воды. Если какая-либо вода попадет в любую бутылку, а затем будет вылита, она исчезнет навсегда. С каким наименьшим количеством воды вы можете начать и при этом выполнить задачу?

Подсказка

Решите диофантово уравнение \ (13x + 20 y = 2 \) (почему?). Затем рассмотрим, какое значение \ (k \) (параметр в решении) является оптимальным.

Как решить систему уравнений с модом?

Ну мод проще в обращении.У нас есть только $ m $ чисел $ \ pmod m $: $ 0,1, \ dots, m-1 $ и уже $ m \ Equiv 0 $ (также $ -1 \ Equiv m-1 $), он идет в цикл точно так же, как часы в день $ \ pmod {12} $.

Точнее, $ a \ Equiv b \ pmod m $ означает $ \ m | (ab) $, и арифметические операции, такие как $ +, -, \ cdot $, очень удобны для него, $ \ Equiv $ действует точно так же, как уравнение.

Вы можете попытаться решить это, например, $ b \ Equiv 22-7a \ pmod {26} $, а затем заменить его на другое, $ 5 \ Equiv -3a + 22 $, так что $ 3a \ Equiv 17 $, но $ \ pmod {26} $, эти $ 17 $ можно заменить, например, на $ -9 $ (потому что $ 17 \ Equiv -9 \ pmod {26} $), а $ 3 $ — это , взаимно простое и 26 $, поэтому можно разделить на $ 3. $.

Поскольку все в $ \ bmod {26} $, вы можете использовать большинство методов для решения других одновременных уравнений. Вместо деления на дроби используйте модульное деление (которое включает алгоритм Евклида).

Например, давайте воспользуемся методом исключения Гаусса для этой задачи. $$ \ begin {align} 12 & = 2a + b \ pmod {26} \\ 15 & = 9a + b \ pmod {26} \ end {align} $$ Вычитание первого из второго дает $$ 3 = 7a \ pmod {26} $$ Используя алгоритм Евклида, мы получаем, что $ 15 \ times7 = 105 \ Equiv1 \ pmod {26} $.Итак, умножив обе части на 15 $, получим $$ 19 = а \ pmod {26} $$ Вычитая 2 доллара, умноженные на второе, из 9 долларов, умноженных на первое, дает $$ 78 = 7b \ pmod {26} $$ Поскольку $ 78 \ Equiv0 \ pmod {26} $, умножение обеих частей на $ 15 $ дает $$ 0 = b \ pmod {26} $

Использование алгоритма Евклида-Уоллиса

Как описано в этом ответе, мы можем использовать алгоритм Евклида-Уоллиса для инвертирования $ 7 \ bmod {26} $: $$ \ begin {array} {rrrrrrr} && \ color {оранжевый} {3} & \ color {оранжевый} {1} & \ color {оранжевый} {2} & \ color {оранжевый} {2} \\ \ hline \ color {# 00A000} {1} & \ color {# 00A000} {0} & 1 & -1 & \ color {red} {3} & \ color {blue} {- 7} \\ \ color {# 00A000} {0} & \ color {# 00A000} {1} & -3 & 4 & \ color {red} {- 11} & \ color {blue} {26} \\ \ color {# 00A000} {26} & \ color {# 00A000} {7} & 5 & 2 & \ color {red} {1} & \ color {blue} {0} \ end {массив} $$ Это означает, что $ 3 \ times26-11 \ times7 = 1 $, что означает, что $ -11 \ times7 \ Equiv1 \ pmod {26} $.

Добавить комментарий

Ваш адрес email не будет опубликован. Обязательные поля помечены *